SlideShare a Scribd company logo
1 of 23
COMPREHENSIVE         “พยายามรวมเท่าที่ทาได้ อะ… จริ งๆ ลองดูของปี
                 2008ด้ วยละกันนะคล้ ายกันอยู่เหมือนกัน”

    TEST              Thanks เพื่อนๆ ที่ช่วยจาข้ อสอบออกมา ทั ้ง Rama
                 และ PI นะค้ าบบ , เพื่อนๆ ที่ช่วยเราแยกข้ อสอบตาม ward



RAMA & PI 2010




                                                                      1
MEDICINE                                                               A. add lamivudine                        B. add interferon-alpha
   (Med-Neuro) 64 years-old Male with history of drinking 4 bottle4                   C. add steroid                           D. หยุด diclofenac
    of beer for 10 years. Presents with 4 episodes of 1-2 mins of                      E. advice splenectomy
    Generalized Tonic Clonic seizure. Between each attack patient
    gains consciousness completely. PE:Confusion, Signs of chronic                    (MED-GI) หญิงอายุ 30 ปี มาตรวจสุขภาพ ผลปรากฏว่า HBsAg + ,
    liver disease, resting tremor without flapping tremor, Normal                      HBsAb -, ผล liver function test ปกติ ควรทาอย่างไรมากทีสด
                                                                                                                                             ่ ุ
    Neuro exam.
    What is an immediate Mx?                                                           A. รักษาด ้วย interferon                B. ให ้ HBV vaccine
    1. Load Oral Hydantoin                                                             C. ตรวจ U/S abdomen                     D. ตรวจ AFP
    2. Load IV Hydantoin                                                               E. ตรวจ HBsAg ซ้าในอีก 6 เดือน
    3. Load IV Benzodiazepine
                                                                                      (MED-GI) ชายไทย 45ปี ปวดใต ้ชายโครงขวามา 5 วัน 10วันpta มี
    4. Load IV Barbidurate
                                                                                       อาการถ่ายเป็ นมูกเลือด v/s 38 c P 80 RR 16 BP 120/80     PE.
    5. Treat Hepatic encephalopathy
                                                                                       พบคลาตับได ้ 3 cm . กดเจ็บ มี jaundice จะพิจารณาให ้ ATB อะไร
   (Med-Neuro) 60 years-old woman presents with right drooping                        1.   Cloxacillin
    eyelid for 3 months. There is fluctuation of symptom. Rt. Eyelid                   2.   Cotrimazole
    almost normal in the morning. On examination – Moderate                            3.   Quinolone derivertive
    drooping Rt. Eyelid, Full EOM, Equal pupil, Normal muscles power,                  4.   Nitronidazole derivertive
    Normal DTR
    What is the most likely diagnosis?                                            (Med-ID)หญิงอายุ 45 ปี U/D เป็ น SLE on steroid มาด ้วยเริมมี
                                                                                                                                             ่
    1.senile ptosis                                                                ไข ้ ไอ หายใจลาบาก CXR (1 mo ago) พบ RLL mass size 1*1
    2.MG                                                                           cm F/U CXR (ครังนี้) พบว่า mass มีขนาดใหญ่ขน (จาขนาด
                                                                                                     ้                             ึ้
    3.Horner’s syndrome                                                            ไม่ได ้) ตรวจร่างกาย มี questionable stiffness of neck LP พบ WBC
    4.CNIII palsy                                                                  เป็ น mononuclear cell จานวน 65, Glucose 65(blood sugar 100),
    5.Oculopharyngeal muscular dystrophy                                           Protein 45 ถามว่าเกิดจาก?
                                                                                   a) Penicillosis
   (Med-Hemato) หญิง 50 ปี ปวดสะโพกขวา 3 เดือน ตรวจร่างกายมีซด,        ี          b) Histoplasmosis
    limit ROM right pelvis due to pain, X-ray มี osteolytic lesion ที่ right       c) Candidiasis
    pelvis involve acetabulum และมี scattered osteolytic lesion ที่ skull          d) Aspergillosis
    PBS มีรูปให ้ดู ดูเป็ น Rouleaux formation                                     e) Cryptococcus
    Diagnostic investigation ทีเหมาะทีสดคือ
                                  ่      ่ ุ
    1.Serum Ca, PO4 level                                                         (Med-ID)ผู ้ป่ วยท ้องเสียเรือรัง ตรวจอุจจาระพบตามรูป เป็ น capillaria ควร
                                                                                                                ้
    2.Protein electrophoresis                                                      ป้ องกันยังไง?
    3.MRI pelvis                         4.Bone scan                               ก.      ล ้างมือก่อนรับประทานอาหาร
                                                                                   ข.       กินหอยสุก
   (Med-GI)ผู ้ป่ วยชาย 60 ปี มาด ้วย easy bruising 2 wk มีประวัตใช ้
                                                                  ิ
                                                                                   ค.      กินปลาสุก
    diclofenac for low back pain for several years ตรวจร่างกายพบ small
                                                                                   ง.      ใส่รองเท ้า
    ecchymosis upper leg, mild hepato-splenomegaly, Lab : Hb 11
    g/dL, MCV 100, WBC 4,100, Plt 95,000,
    LFT : AST 37, ALT 150, ALP 130, total bilirubin/direct bilirubin           
    1.2/0.4, albumin 30, globulin 40 g/L, HbsAg +ve รักษาอย่างไร

                                                                                                                                                           2
   (Med-ID)ชาย 40 ปี มีอาการไอเรือรัง น้ าหนักลด ไข ้ตาๆ chest x-ray มี
                                    ้                  ่                          C. Neutrophil
    cavity at LUL, AFB 1+ ได ้รักษาเป็ น IRZE 2 เดือน อาการไม่ดขน ตรวจ
                                                               ี ึ้               D. Complement
    AFB 2+ จะทาอย่างไรต่อ
    A. ยาเดิมและนัดติดตามผลการรักษา 1 เดือน                                       (MED-ID) ผู ้ป่ วยชายอายุ 15 ปี ปั สสาวะลาบากมา 1 วัน เมือ 3 วันก่อนมี
                                                                                                                                             ่
                 ้                             ้
    B. ส่งเพาะเชือและให ้ยาเดิมจนกว่าผลเพาะเชือจะออก                               เพศสัมพันธ์กบหญิงขายบริการโดยไม่ได ้ป้ องกัน ตรวจร่างกายพบ
                                                                                                 ั
    C. เปลียนยาใหม่
           ่                                                                       yellowish discharge per urethral meatus ส่ง G/S เป็ น gram
                   ้
    D. ส่งเพาะเชือและเปลียนยาใหม่
                          ่                                                        negative intracellular diplococci with numerous PMN รักษาด ้วยยา
    E. ยาใหม่(second line drug)... ตัวและยาเดิม                                    ใด
                                                                                      A. Doxycycline oral               B. Azithromycin oral
   (MED-ID)มีไข ้ ปวดหัว มา 1 สัปดาห์ 3 วันก่อน มีตาเหลือง ปั สสาวะออก               C. Ceftriaxone IM                 D. Benzathine pennicilin G IM
    น ้อยลง PE: mild pale, mild jaundice, mild hepatosplenomegaly,
    muscle tender                                                              (MED-ID)ชายอายุ 16ปี มาด ้วยปวดศีรษะ และมีไข ้ ตรวจร่างกาย BT 39
         LAB: CBC: HB 9.5, WBC 15400(N79%, L20%, E3%)                           stiffness of neck, Organism?
         BUN/Cr=95/4
                                                                                    a. S. aureas
         UA: sp 1.015, pH 6.5, RBC 10-20, WBC 2-3
         LFT: AST 210, ALT 150, ALP 71, TB 7.0, DB 5.2, TP7.2, ALB                  b. N. gonorrheae
         3.8 ให ้ DX                                                                c. H. Influenzae
    A. Malaria
    B. Scrub typhus                                                            (MED ID) ผู ้ชาย 25 ปี มีไข ้ ต่อมน้ าลายโต บวม กดเจ็บ กด Stensen’s
    C. Leptospirosis                                                            duct มี yellow discharge จึงส่ง gram stain
    D. Acute viral hepatitis                                                    ให ้รูปมา เป็ น Gram-negative rod with a characteristic "safety pin"
                                                                                appearance (bipolar staining)
    (MED-ID) ผู ้ป่ วยชาย 40 ปี ทางานช่างเรียงพิมพ์ มีอาการไอเรือรัง 2
                                                                 ้
                            ้
     เดือน ตรวจเสมหะพบเชือวัณโรค จะได ้ยารักษาวัณโรค 4 เดือน ควรทา                     A. Neisseria meningitides B. Bacteroides fragilis
     อย่างไร                                                                           C. Pseudomonas aeruginosa D. Burkholderia pseudomallei
    A.   พักงาน 2 สัปดาห์                                                      (MED ID)กรวยไตอักเสบได ้ ceftriaxone 7วัน หลังจากนันถ่ายเหลวเป็ น
                                                                                                                                   ้
    B.   ใส่หน ้ากากทางาน 2 สัปดาห์
                                                                                                  ้
                                                                                น้ า5ครังคิดถึงเชืออะไร
                                                                                        ้
    C.   ย ้ายงานไปอยู่ในทีทมอากาศถ่ายเทสะดวก
                              ่ ี่ ี
    D.   ติดหลอดไฟทีมแสง UV
                          ่ ี
                                                                                       C.difficile
    E.   ให ้ผู ้ร่วมงานกิน INH prophylaxis 3 เดือน
                                                                                  (MED-cardio) ผู ้ป่ วยชาย 50 ปี เหนื่อยง่าย นอนราบไม่ได ้ มา1เดือน
                                                                                   ตรวจร่างกาย BP 130/80 , RR 24, PR 80 PMI 6th ICS anterior
    (MED-ID) ผู ้ป่ วยหญิงอายุ 30 ปี
                                    2 เดือนก่อนมารพ. ปวดข ้อมือ ข ้อเท ้า
                                                                                   axillary line , no mur mur       Lung : fine crepitaion at basal both
      ได ้ยารับประทานดีขน 2 สัปดาห์กอน มีไข ้ เจ็บหน ้าอกซ ้ายเวลาหายใจ ไม่
                        ึ้          ่                                              lung ยาใดทีจากการศึกษาวิจัยว่าช่วยลด mortality rate
                                                                                                ่
      ไอ ไม่หอบ ตรวจร่างกาย: mild pale conjunctiva, swelling
      interphalanx, decreased breath sound LLL. LAB: CBC Hb10, WBC                A.   Atenolol
      4800(N80%, L20%), Plt 160000 ถาม pathogenesis ว่ากลไกใดเสีย                 B.   Digoxine
     A. Antibodies                                                                C.   Diltiazem
    B. Macrophage                                                                 D.   Amiodipine
                                                                                  E.   Bisoprolol

                                                                                                                                                        3
B. IV furosemide
   (Med-Hemato)ผู ้ป่ วย femur หักผ่าตัด bleed ไป 2000 ml เลือดgr. AB             C. IV Nifedipine
    แต่รพเลือด gr. AB หมด จะให ้ blood component อะไร                              D. IV propanolol
    A whole blood gr A
    B whole blood gr O                                                            (Med-Cardio)EKG-low voltage QRS manage ยังไง
    C PRC gr A                                                                     ก.   Pleural tapping
    D PRC gr B                                                                     ข.    Cardiothoracotomy
    E PRC gr O                                                                     ค.   Cardiocentesis

   (MED-hemato) ชาย 30 ปี chronic alcohol drinker for 10 yr มาด ้วย              (MED Cardio)หญิงอายุ 40 ปี แข็งแรงดีตลอด มาด ้วยเรืองlow grade
                                                                                                                                      ่
    อ่อนเพลีย 3 เดือน ตรวจร่างกายพบ pale conjunctiva, anicteric,                   fever และเหนื่อยง่ายมา 14 days, PE : murmur gr.3/6 at apex CBC:
    glossitis, no hepatosplenomegaly, CBC : Hct 28.5% MCV 125 MCH                  normal wbc, Echo : Oscillating mass 0.7*0.9 cm at mitral valve
    28 MCHC 34 WBC 9,200 (N 60 L 30 B 4 E 1) Plt 90,000 What is                    MOST likely diagnosis ?
    the cause of his symptoms?                                                      A. MV prolapsed
                                                                                    B. Subacute IE
    a. Chronic alcoholism     b. Folate deficiency      c. Iron deficiency          C. Rheumatic heart disease
    d. Aplastic Anemia        e. MDS
                                                                                    D. Cancer endocarditis
                                                                                    E. Acute IE
   (Med-Cardio) ผู ้ป่ วยมา ER คลาชีพจรไม่ได ้ วัด BP ไม่ได ้ ทา EKG (มีรูป
    ให ้ดู) เป็ น VT จะทาอะไรก่อน                                                 (MED Cardio)Female, 50 years old, substernal chest pain
    A adenosine                                                                    PE: BP 150/110 mmHg, PR 100/min, mild dyspnea,H&L : WNL
    B defibrillator                                                                EKG: ST elevation in II,III,aVF
    C electrocardioversion                                                         Question: After ASA giving – Next step of management?
    D amiodarone
    E adrenaline                                                                   a.   Enalapril
                                                                                   b.   Nifedipine
   (Med-Cardio)หญิงอายุ 60 ปี เหนื่อยเวลาทางานบ ้าน ต่อมาเหนื่อยมาก
                                                                                   c.   Propanolol
    ขึนเรือยๆ
      ้ ่
                                                                                   d.   Furosemide
    BP 100/80, RR 30/min, PR 100/min regular, small volume, slow up
                                                                                   e.   Isosorbide dinitrate
    stroke carotid pulse, PMI at 5th ICS, 3 cm lateral to MCL, normal
    S1, decrease S2, bilateral fine crepitation
    ผู ้ป่ วยน่าจะเป็ นโรคอะไรมากทีสด่ ุ                                          (Med-Endo)ผู ้ป่ วยชายอายุ 22 ปี ปั สสาวะบ่อยมา …. วัน ปากแห ้ง จึงดืม
                                                                                                                                                        ่
            A. AS                                                                  น้ ามาก ๆ ไม่ปวดหัว ไม่มตามัว น้ าหนักเท่าเดิม จะส่งตรวจอะไรเพือช่วยใน
                                                                                                           ี                                      ่
            B. MS                                                                  การวินจฉั ย (โจทย์มเท่านี้จริงๆ)
                                                                                          ิ            ี
            C. constrictive pericarditis
                                                                                   A.   Urine sugar
   (Med-Cardio)ผู ้ป่ วยชาย อายุ 60 ปี มี sudden onset ของ chest pain             B.   Urine specific gravity
    BP แขนซ ้าย 74/55 แขนขวา 170/110 PR 60/min จะ management                       C.   Plasma glucose
    อย่างไร                                                                        D.   Serum Creatinine
    A. oral Enalapril                                                              E.   Serum electrolyte

                                                                                                                                                       4
                                    ่
    (Med-Endo)หญิงอายุ 55 ปี ใจสัน ร ้อนง่ายมา 6 เดือน ตรวจร่างกายPR            (Med-Skin) ผู ้ป่ วยมีตมคล ้ายลมพิษเป็ นๆหายๆมา 3 เดือน 1 wk ก่อน มี
                                                                                                        ุ่
    100/min , solitary thyroid nodule ตรวจthyroid scan เป็ น hot thyroid         ตุมน้ าใสขึน PE พบ multiple vesicle (bullous) diameter 2-4 cm, on
                                                                                   ่        ้
    nodule ผู ้ป่ วยปฏิเสธการผ่าตัด นอกจากจะให ้PTUแล ้วจะให ้อะไรอีก            erythematous plague on trunk and upper thigh ถามว่า Dx อะไร
    a. Diazepam                                                                  A urticarial valgaris
    b. I131                                                                      B Bullous pempigoid
    c. Propanolol                                                                C Pempigous valgaris
    d. Lugor solution                                                            D
    e. Methymazole                                                               E TEN

                                                                                (Med-Skin)ผู ้ป่ วยหญิงอายุ 25 ปี มาด ้วยผืนแดงขึนบริเวณหน ้าผากและ
                                                                                                                            ่     ้
   (MED-endo) หญิงอายุประมาณ 60 ปี (มัง) ซึมลง ไม่กนมา 2 สัปดาห์ มี
                                          ้           ิ                          ลามเป็ นตุมน้ าใสอย่างรวดเร็วมา 1วัน ตรวจร่างกายพบผืนแดงและตุมน้ าใส
                                                                                           ่                                            ่        ่
                           ้
    ประวัตปวดเข่าเรือรัง ซือยากินเองตลอด ตรวจร่างกาย Drowsiness,
          ิ         ้                                                            บริเวณหน ้าผากและเปลือกตาซ ้ายบวมแดง ผู ้ป่ วยมีโรคประจาตัวอะไร
    Motor เท่ากัน 2 ข ้าง, BP 140/80 ท่านอน, 100/70 ท่านั่ง, PR 80/min
    ตรวจเลือด plasma glucose 140, Na 115, K ปกติ(จาค่าแน่นอนไม่ได ้),            a. HIV
    Cl 85, CO2 20,                                                               b. SLE
    BUN ปกติ, Cr 1, Serum osmol 240, Urine osmol 500, Urine Na 40
    จะส่งตรวจอะไรต่อ                                                            (Med-Skin)ผู ้ป่ วยชาย 65 ปี มาด ้วยผืนคล ้ายผืนลมพิษเป็ นๆหายๆมา 3
                                                                                                                       ่        ่
                                                                                 เดือน ตรวจร่างกายพบ tense bleb 2-4 cm on erythematous plaque
         A. Urine VMA                                                            วินจฉั ยอะไร
                                                                                    ิ
         B. Serum cortisol                                                            A. urticarial vasculitis                B. bullous pemphigoid
                                                                                      C. pemphigus vulgaris                   D. erythema multiforme
   (MED-Endo)ชายอายุ 45 ปี ตรวจพบว่าเป็ น DM มา3 เดือน FBS 220,                      E. toxic epidermal necrolysis
    HbA1c 8.5 ,TG 230 ,choles 210 , น้ าหนัก80 Kg , สูง165 Cm จะstart
    ยาเบาหวานอะไร                                                               ผู ้ป่ วยชาย อายุ 20 ปี คันบริเวณง่ามนิวมือ ขูดผิวหนังไปตรวจด ้วยกล ้อง
                                                                                                                        ้
    A. Metformin                                                                 จุลทรรศน์กาลังขยายตา พบลักษณะดังภาพ
                                                                                                          ่
    B.   Glybenclamide
    C.   Rapeglenide
    D.   Insulin
    E.   Arclabose


   (MED Endo)ผู ้ป่ วยหญิง 40 ปี มีก ้อนทีคอมา 9 เดือน ก ้อนไม่โตขึน ตรวจ
                                           ่                        ้
    ร่างกายพบ thyroid nodule 3 cm, firm consistency ทาอะไรต่อ
        A. thyroid scan                  B. thyroid uptake
        C. ultrasound                    D. antimicrosomal anyibbody
        E. fine needle aspiration

    (MED Endo)หญิง 16ปี มาด ้วย Abdominal pain(clinical DKA), initial
     MX?
    A. IV 0.9% NaCl
                                                                             การรักษาทีเหมาะสมกับผู ้ป่ วยรายนี้
                                                                                       ่
    B. IV NaHCO3

                                                                                                                                                           5
A.   Antharcin                                                             10 ปี X-Ray ปอดพบ hyperairation, falttern diaphragm, Infiltration
    B.   Sulfur Ointment                                                       ที่ upper lung ทังสองข ้าง
                                                                                                ้
    C.   Benzyl Oxydase
    D.   Coal tar ointment                                                     a. COPD
    E.   Benzylgamma hexachloride                                              b. Pulmonary TB

                                                                              (Med-Pulmo) ชาย40ปี ทางานโรงพิมพ์ ได ้รับการวินจฉั ยว่าเป้ นวัณโรค
                                                                                                                              ิ
   (MED-skin)ชาย 25 ปี มีผนแดง คัน หนา ทีหนังศีรษะ มา...........เดือน
                             ื่             ่                                                                     ่                   ้
                                                                               ปอด ให ้ IRZE 4month จะทาอย่างไรเพือป้ องกันการแพร่เชือให ้คนอืน่
    ไม่มอาการผิดปกติอย่างอืน ตรวจร่างกาย small & large erythematous
        ี                  ่                                                   1.Isolates 2 wks
    scaly plaque at scalp 50 % , yellowish spotting at nail ;Diagnosis ?       2.ใส่ Mask 2 wks
    A. Tinea capitis                                                           3. ให ้ทางานในทีอากาศถ่ายเท
                                                                                                ่
    B. Psoriasis                                                               4.ให ้เพือนร่วมงานกิน INH 3 month
                                                                                        ่
    C. Seborrheic dermatitis                                                   5.ให ้เปลียนหลอดไฟทีทางานเป้ นแสง UV
                                                                                          ่           ่

                                                                              (Med-Pulmo)ชายทางานโรงงานย่อยหิน ไอเรือรัง เหนื่อยง่าย มา 1 ปี มี
                                                                                                                     ้
                                                                               cxr มาให ้ diag
   (Med)ทหารอายุ 20 ปี ถูกนาส่ง ER เนื่องจากเป็ นลมขณะกาลังฝึ กทีกลาง
                                                                  ่            ก.    TB
    แดดเป็ นเวลานาน ตรวจร่างกาย T 42 , PR 130 , RR 30 , BP 140/90              ข.     Silicosis
    mmHg. unconscious and occasional seizure What is the risk factor           ค.    Sarcoidosis
    for this patient?
                                                                              (Med-Pulmo)ชายอายุ 20 ปี มาด ้วย suddenly develops chest pain
    1. Male gender                                                             and progressive dyspnea within 3 hours. PE: decreased breath
    2. High humidity                                                           sound on the left lung and increased tympany on percussion
    3. Previous seizure                                                        ถาม emergency management?
    4. young age group                                                         a) ET tube intubation
                                                                               b) Thoracocentesis
    5. genetic predilection
                                                                               c) Oxygen support
   ชาย50 ปี underlying HT มาด ้วยปวดทั่วๆ ท ้องมา1วัน ขาไม่มแรงเดิน
                                                             ี                 d) ICD
    ไม่ไหว                                                                     e) จาไม่ได ้อ่ะ
    ตรวจ PE PR arm = 110 pulse ทีขาคลาไม่ได ้ BP 90/60
                                    ่
    What is initial management                                                (Med-Pulmo)ผู ้ป่ วยชาย อายุ 70 ปี มีอาการเหนื่อยง่ายเวลาขึนบันได
                                                                                                                                          ้
           A. Emergency sugery                                                 เป็ นๆ หายๆ มา 10 ปี มีไอมีเสมหะ สูบบุหรีวนละ 1 ซอง มา 30 ปี ตรวจ
                                                                                                                        ่ ั
           B. Dopamine                                                         ร่างกายพบ
           C. IV resuscitation                                                 Lung : Barrel shape, decrease breath sound both lungs.
           D. Whole blood                                                                           ่ ่
                                                                               ข ้อใดเป็ นการรักษาทีชวยให ้คุณภาพชีวตดีขน
                                                                                                                     ิ    ึ้
           E. Cut down วัด CVP                                                 A. Bronchodilator
                                                                               B. Ipratropium bromide
                                                                               C. Inhalation corticosteroid
   (Med-Pulmo) ผู ้ป่ วยชายอายุ 55 ปี มาด ้วยอาการเหนื่อยง่ายมากขึนเวลา
                                                                   ้           D. Pulmonary rehabilitation
    ทางาน มีไอเล็กน ้อย ทางานทีโรงงานทาครกหิน สูบบุหรี่ วันละ 1 ซองมา
                                 ่                                             E. Long term oxygen therapy


                                                                                                                                                    6
   (MED Pulmo)ชายอายุ 35 ปี ทางานอยู่ในโรงงานเกียวกับแร่ใยหิน มี
                                                   ่                         (MED Rheumato)ญ อายุ 40 ปี มีอาการปวดช ้อมือ ข ้อนิว ทัง 2 ข ้าง รู ้สึก
                                                                                                                                   ้  ้
    อาการไอเรือรังมาประมาณ 1 ปี chest x-ray พบ diffue infiltration ที่
               ้                                                              เหมือนมีไข ้ตาๆ ตรวจร่างกาย มี Tender&Swelling ที่ 2nd MCP,2nd
                                                                                           ่
    ปอด 2 ข ้าง จงให ้การวินจฉั ย
                            ิ                                                 3rd 4th PIP ข ้างขวา และ 3rd MCP 3rd 4th PIP ข ้างซ ้าย

    A. Silicosis                                                               .....
    B. Aspergillosis                                                         (Med-Toxico)ชาย 20 ปี พีสาวพามามาที่ ER ด ้วยปั ญหายาเสพย์ตด มี
                                                                                                      ่                                       ิ
    C. Asbestosis                                                             อาการปวดกระดูก และกล ้ามเนื้อ ตัวงอร ้องไห ้ ยอมรับว่าตนเองใช ้สาร
                                                                              เสพย์ตดหลายชนิดปนกัน ตรวจร่างกาย Temp 37 PR 87 RR 20 ถาม
                                                                                     ิ
   (MED Pulmo)A previously healthy 60-year-old man presents with             Manageไง
    left-side substernal pain then productive cough with rusty                1. Oral clonidine
    sputum, no history of smoking and he denies HIV risk, chest pain          2. Oral naltrexone
    is sharp, locate on left, no radiate PE: T 39C RR 26/min BP 90/60         3. Oral methadone
    P90 Chest: dullness on percussion and decrease breath                     4. IM Haloperidol
    sound(โจทย์ตรงตรวจร่างกายมันมีอก แต่มแต่พวก negative finding อีก
                                       ี      ี                               5.IV Naloxone
               ่
    2 อย่าง ซึงจาไม่ได)้ ให ้ Dx
    A. Pericarditis                                                        (Med-Toxico)ชาย 35 ปี โดนงูกดทีมอขณะตัดกิงไม ้ทีสวนยางพารา 3
                                                                                                                     ั ่ ื           ่   ่
    B. TB pleuritis                                                           ่
                                                                            ชัวโมงก่อนมารพ. มีปวดบริเวณมือถึงแขน PE: ecchymosis and bullous
                                                                            ทีมอ ถามว่างูชนิดใดทาให ้เกิดอาการ
                                                                                ่ ื
    C. Influenza pneumonia
                                                                                            a) งูเห่า
    D. Obstructive pneumonitis                                                              b) งูกะปะ
    E. parapneumonic effusion                                                               c) งูแมวเซา
                                                                                            d) งูสามเหลียม  ่
   หญิง 60 ปี underlying DM, HTN, DLP มารพ.ด ้วยอาการ Dyspnea and                          e) งูทับสมิงคลา
    Diaphoresis . What is the most likely diagnosis ?                      (MED-toxico) ผู ้ป่ วยหญิง 20 ปี ทะเลาะกับเพือนชาย น ้อยใจจึงกินน้ ายา
                                                                                                                                   ่
    A. Panic attack                                                          ล ้างห ้องน้ า เพือนชายพามารพ.ทันที เอาขวดน้ ายาล ้างห ้องน้ ามาด ้วย
                                                                                               ่
    B. Pulmonary embolism                                                    พบว่าประกอบด ้วย HCl 15% vital sign ปกติด ี มี mild erosionทีปากและ ่
    C. Acute myocardial infarction                                           กระพุ ้งแก ้มเล็กน ้อย ผู ้ป่ วยรู ้สึกตัว บ่นว่าปวดแสบในช่องปากและลาคอ
    D. Pneumonia                                                             กลืนน้ าลายไม่ได ้และปวดท ้อง ถาม ข้อห้าม การรักษาในผู ้ป่ วยรายนี้ คือ
                                                                             ข ้อใด
   (MED-nephro) ชาย 50 ปี เพลีย คลืนไส ้ อาเจียน ปั สสาวะน ้อยลง 2 วัน
                                     ่
                                                                                           a) ดืมน้ ามากๆ
                                                                                                 ่
    ตรวจร่างกายพบว่ามี dyspnea, edema, lungs-rales BLL, BUN 80 Cr
                                                                                           b) กิน sucralfate
    5.7 นอกจาก UA แล ้วจะ investigate อะไร
                                                                                           c) กิน paracetamol
                                                                                           d) กินสารละลาย NaHCO3
    a. Plain KUB    b. U/S Kidney c. Renal Scan d. IVP                                     e) ส่ง endoscope ใน 48 ชัวโมง      ่

   (MED-Rheumato) หญิง 25 ปี มีอาการปวดข ้อนิว เป็ นมากตอนเช ้า มี
                                                 ้
    painless oral ulcer, DCT-positive. Diagnosis?

    a. Reiter b. Reactive Arthritis    c. SLE d. Rheumatoid Arthritis


                                                                                                                                                    7
   (MED Tox)ชายอายุ 26 ปี ญาตินาส่ง รพ.ด ้วย หมดสติ ปลุกไม่ตนมา 2
                                                              ื่         (MED)ผู ้ป่ วยชาย 20 ปี มาด ้วยท ้องโต ขาบวมมา 3 เดือน ตรวจร่างกายพบ
    ชม สงสัยว่ากินยาเกินขนาด PE: semicoma, Pupil dilate 4 mm both         sign chronic liver disease +ve, ascites 2+, mild jaundice Lab : Hct
    eyes, ปากแห ้ง, ผิวแห ้ง, decreased bowel sound EKG: wide QRS,        28%, WBC 3,500, AST 230, ALT 100, ALP 30, total protein 80 g/L,
                                                                          albumin 30 g/L, anti-HBc +ve, HbsAg –ve, anti-HBs –ve, anti-HCV
    Prolong QT คิดว่าน่าจะเกิดจากยาใด
                                                                          –ve, ANA 1:40
    a) codeine                                                              A. serum ferritin           B. protien electrophoresis
    b) hyoscine                                                             C. HCV PCR                  D. anti-smooth muscle antibody
    c) amitryptyline
    d) amphetamine                                                         (MED) หญิงไทย60ปี เป็ นCA Stomach ระยะสุดท ้าย แพทย์ไปเยียม   ่
    e) CPM                                                                  บ ้าน พบว่าผอมมาก หนังหุ ้มกระดูก ญาติบอกผู ้ป่ วยทานอาหารได ้น ้อย
                                                                            มาก ขอปรึกษาเรืองการให ้อาหารกับผู ้ป่ วย ข ้อใดเหมาะสมสุด
                                                                                              ่
   (MED TOX) ทหารทีชายแดนภาคใต ้ปะทะกับผู ้ก่อการร ้ายทุกวัน มี
                      ่                                                         1.Oral feeding as tolerance
    อ่อนเพลีย ซีด ปวดท ้อง ตรวจร่างกายพบว่ามีแถบสีดาทีเหงือกด ้านบน
                                                      ่                         2.No feeding
    เกิดจากสารพิษใด                                                             3.Jejunostomy tube feeding
       A.ปรอท                B. ตะกัว
                                    ่                                           4.partial parenteral nutrition
       C.ทองแดง              D.แมงกานีส                                         5.Total parenteral nutrition
       E.คาร์บอน
                                                                           (MED-ID)หญิง อายุ 50 ปี ทาสวน ปวด คันข้ อเท้ า ตรวจร่างกายพบ erythematous,
                                                                            tubular lesion เกิดจากเชื ้อใด
   (MED) ผู ้หญิง แข็งแรง no U/D มาด ้วยปวดขาขวามา 5 วัน, กินยาคุมมา
    3 เดือน ตรวจร่างกาย swelling and tender Rt. leg ให ้ Dx                        1.   Toxocara
               a) cellulitis                                                       2.   Trichinella
               b) lymphedema                                                       3.   Stongyloides
               c) deep vein thrombosis                                             4.   Gnathostomiasis
               d) osteolitis                                                       5.   Angiostrongyrus
               e) arterial occlusion
                                                                           (MED)ผู้ ป่วย post op. cardiac surgery มีอาการสับสนโวยวาย เห็นพญายม จา
 (MED )หญิงอายุ 45 ปี มาด ้วย central upper abdominal pain 2 days
                                                                            ญาติไม่ได้ พูดว่าอย่ามาหลอกกันเลย what is immediate management
  ,PE:fever, tender epigastrium, Past history: no smoking,no
  trauma,no alcoholic , LFT: AST 146,ALT 160,TB 3.0 DB1.9;
                                                                                   A.   Diazepam
  Amylase 800, CXR: WNL Most appropriate investigation?
                                                                                   B.   Midazolam
                                                                                   C.   Fluoxetine
    A.   U/S upper abdomen
                                                                                   D.   Haloperidol
    B.   MRI
                                                                                   E.   Nortriptyline
    C.   CT
    D.   ERCP




                                                                                                                                                     8
   (MED) ชายอายุ 70ปี clinical dementia ,มีอาการหลงลืม กินยาเกินขนาดเพราะจา                  2 Corneal ulcer ด้ านซ้ าย
    ไม่ได้ ว่ากินไปแล้ ว no neuro deficit ถามว่าmost factor ที่จะทาให้ อาการแย่ลงคือ          3 ชาหน้ าซีกซ้ าย
    อะไร                                                                                      4 Sensorineural hearing loss หูซ้าย
                   A. ไขมันในเลือดสูง                                                         5 ชาลิ ้นด้ านหน้ าซ้ าย
                   B. เส้ นเลือดสมองตีบ
                                                                                             (MED)ชายไทยอายุ 50 ปี เป็ น   Hypertension 5 ปี   ได้ รับยา atenolol มาด้ วย
                   C. อาชีพก่อนเกษี ยณ
                                                                                              sudden onset chest pain วัด BP right arm 140/90 , left arm 70/40          ให้ ยา
                   D. ความเครียดของผู้ดแลู
                   E. ความสัมพันธ์ ของคนในครอบครัว                                            รักษาตัวใดอันดับแรก (ตอบข้ อ 2)

                                                                                                              A.   sublingual nitroglycerine
   (MED)ผู้ ป่วยชายอายุ 50 ปี เหนื่อยหอบ ตัวบวม ปั สสาวะออกน้ อย มาสองวัน       urine
                                                                                                              B.   nitropusside IV
    output 50 Bun/Cr 50/5.6 CXR pulmonary congestion นอกเหนือจาก                                              C.   propanolol
                                                                                                              D.   sublingual nifedipine
    UA   ให้   Initial investigation เพิ่มเติม
                                                                                         Pediatrics
               A.   Film KUB
               B.   Ultrasound KUB                                                                                                        ้
                                                                                             (Ped-Hemato) เด็กชาย 3 ปี เลือดออกในข ้อบ่อย ชาง่าย มีพชายและลุง
                                                                                                                                                    ่ี
                                                                                             มีอาการคล ้ายกัน CBC ปกติ Plt 170000,APTT prolong, bleeding time
               C.   Renal scan
                                                                                             ปกติ, PT ปกติ,
               D.   IVP                                                                      TT ปกติ Dx?
               E.   CT abdomen                                                               - Hemophilia
                                                                                             - Hereditary thrombocytopenia
   (MED)หญิงอายุ 55 ปี ปวดเข่ามา 2 ปี ตรวจร่างกายไม่พบ joint deformity ชัดเจน               - Hereditary platelet dysfunction
    แต่พบใบหน้ ากลม มี trunkal obesity, supraclavicular fat pad สาเหตุที่ทาให้ เกิด          - Von Willebrand disease
                                                                                             - Congenital factor VII deficiency
    อาการข้ างต้ น คือ
                    a. Adrenal adenoma                                                      (Ped-ID)เด็กหญิง 14 ปี ปั สสาวะแสบขัด คันช่องคลอดและทวารหนักมา
                    b. pituitary microadenoma                                                1 เดือน ไม่เคยมีเพศสัมพันธ์ ตรวจร่างกาย erythematous rash at
                    c. pituitary macroadenoma                                                vagina & perigenital area , yellowish discharge ถามการส่งตรวจทีม ี
                                                                                                                                                            ่
                    d. ectopic ACTH                                                          ประโยชน์ในการวินจฉั ยโรค
                                                                                                               ิ
                    e. exogenous glucocortocoid                                              A. urine culture
                                                                                             B. scottape technique
   (MED)ผู้ ป่วยชายอายุ 40 ปี มาด้ วย ปากเบี ้ยวด้ านซ้ ายและตาซ้ ายปิ ดไม่สนิทมา 1         C. darkfield microscopic
                                                                                             D. discharge Gram stain
    สัปดาห์ clinical ที่พบร่วมได้ ในผู้ป่วยรายนี ้คือ                                        E. discharge KOH preparation
    1 แขนขาขวาอ่อนแรง

                                                                                                                                                                             9
     (PED ID)หญิง 15 ปี มีไข ้ ไอแห ้งๆ ตรวจร่างกาย BP 100/60mmHg, PR                     C.   ให้ ซกผ้ าปูที่นอนเองทุกครั ้งที่ปัสสาวะราด
                                                                                                     ั
      100/min, T 37.9c, RR 25/min fatigue, malaise, nasal congestion,                      D. Imipramine
      mild injected pharynx and tonsils, cervical lymph node โต
      a. influenza                b. acute sinusitis                                       E. พบจิตแพทย์
      c. pneumonia                d. pharyngitis
      e. infectious mononucleosis
                                                                                          เด็กสามขวบ ไม่มเขียว พัฒนาการปกติหมด หนัก 14kg สูง 95cm ข ้อใด
                                                                                                           ี ้
     (Ped-Dev) เด็ก 1 ขวบ 6 เดือน แม่พามาเรืองพูดช ้า ยังไม่พูดคาทีม ี
                                               ่                    ่                      ถูกต ้อง
      ความหมาย เรียกไม่คอยหัน ชอบดูทววนละ 2-3 ชม. ชอบเล่นคนเดียว ไม่
                          ่            ี ี ั                                               a. ricket
                 ี้
      สบตา ไม่ชบอกความต ้องการ แม่เป็ นคนเลียงเอง Dx?
                                             ้                                             b.hypopituitary
      - Selective mutism                                                                   c.hypothyroid
      - Hearing impairment                                                                 d. …….dysplasia
      - Autism spectrum disorder                                                           e.normal variant
      - Inappropriate childbearing
      - Developmental language disorder                                                   (PED-newborn) Newborn, preterm, low birth weight มีอาการเหนื่อย
                                                                                           O2 sat 86% PaO2 58 mmHg, plain film พบ ground glass both lungs
     (PED-dev) มารดาพาลูกชายอายุ 8 ปี มาปรึกษาท่านด ้วยปั ญหาการเรียน                     ถาม Management
      และพฤติกรรม โดยเล่าว่า ผู ้ป่ วยมีอาการหงุดหงิดง่าย เกรดเฉลีย 2.8 วิชา
                                                                        ่
      ภาษาไทยได ้เกรด 0 วิชาอืนได ้เกรด2-3 ครูภาษาไทยบอกว่าผู ้ป่ วยไม่
                               ่                                                           a. PPV      b. corticosteroid c. mechanical ventilation d.CPR
      ตังใจเรียน แต่ครูวชาอืนบอกว่า ผู ้ป่ วยตังใจเรียนดี ผู ้ป่ วยทาการบ ้านวิชา
        ้               ิ   ่                  ้
      คณิตศาสตร์ได ้จนเสร็จ แต่ทาการบ ้านภาษาไทยไม่ได ้ อ่านหนังสือไม่ได ้
      ความสัมพันธ์กบเพือนปกติ พัฒนาการปกติ diagnosis is..?
                     ั    ่                                                               (PED-newborn) เด็ก 2800g. ยาว 48 cm. มี cleft lip and palate,
                                                                                           bilateral cryptorchidism, penile length 1 cm, glucose 30. What is
      a. language disorder            b. learning disorder                                 diagnosis?

      c. conductive disorder          d. ADHD                                              a. hyperinsulinemia b. hypopituitarism c. adrenal insufficiency
                                                                                           d. growth hormone deficiency e. inborn error of metabolism
      (PED-dev) at well-baby clinic, 18-month-old baby ร ้องไห ้เวลาไม่ให ้
       กินเอง จับช ้อนกินข ้าวได ้ วาง cubic 4 ชินได ้ หันตามเสียง พูดแต่คาว่า
                                                 ้
       mama. What is the most concerned problem in this children?                         (Ped-Newborn) เด็ก newborn 48 ชม. เหลือง microbillirubin
                                                                                           18 Hct 38%
                                                                                           Aniso2+ Poikilo1+ Fragmented2+ Spherocyte2+ Heinz and
       a. language b. fine motor c. gross motor
                                                                                           Inclusion body negative ถามว่าเกิดจาก
                                                                                           1.ABO incompatibility
       d. behavior         e.social                                                        2.RH incompatibility
                                                                                           3.G6PD
     (PED-dev)เด็ก 7 ปี   ปั สสาวะราดที่นอนมาตลอด กลางคืนปั สสาวะ 3-4 ครั ้ง กลั ้น       4.Hb-H disease
                                                                                           5.Physiological jaundice
       ปั สสาวะได้ 5-10 นาที ตรวจร่างกายปกติ ผลการตรวจปั สสาวะปกติ จะแก้ ไขอย่างไร
      A. ใส่ผ้าอ้ อมสาเร็จรูปตอนกลางคืน
      B. ให้ แม่ปลุกตอนกลางคืนเพื่อปั สสาวะ

                                                                                                                                                           10
   (Ped-Immuno)เด็ก 8 เดือน recurrent bacterial pneumonia มีพชาย    ี่        (PED-pulmo) เด็กอายุ 5 ขวบ 3วันก่อนมีไข ้ ไอ หายใจเร็ว PE:
    เสียชีวตเมืออายุ 9 เดือน ด ้วย recurrent pneumonia ผู ้ป่ วยไม่เคยติด
           ิ   ่                                                                expiratory wheezing, occasional crepitation CXR: mild
       ้
    เชือ virus, fungus ถามว่า ผู ้ป่ วยมีความผิดปดติของ cell ใดในimmune         hyperaeration, parabronchial infiltration ให ้ bronchodilator ไม่ดขน
                                                                                                                                                  ี ึ้
    system                                                                      ให ้ DX
    A. T Cell                                                                    a) Acute epiglossitis
    B. B Cell                                                                    b) Tracheobronchitis
    C. Combine T and B cell                                                      c) Acute Bronchiolitis
    D. Phagocytic cell                                                           d) Bronchopneumonia
                                                                                 e) Acute Asthmatic attack
   (PED-allergy) แม่มลกน ้อย กลัวว่าลูกจะเป็ นภูมแพ ้ เพราะมีประวัต ิ
                        ี ู                       ิ
    ครอบครัวเป็ น allergy จะ advice อะไร                                       (PED-neuro) ผู ้ป่ วยอายุ... มีไข ้ 3 – 4 วัน......... ไม่มี meningeal sign
                                                                                LP : Mononuclear cell 83% Protein 40 % Glucose 60 (Blood
    a. air filter     b. mite-proof mattress        c. no pet                   glucose 80 )

                                                                                1. Brain abscess 2. Viral meningitis
    d. soy milk       e.exclusive breast feeding
                                                                               (Ped-Prevention)เด็ก 9 เดือน ไม่ควรใช ้อะไรเพือป้ องกันอุบัตเหตุ
                                                                                                                              ่             ิ
   (PED-cardio) เด็กหญิงอายุ 6 ปี มีความดันสูง ตรวจร่างกายพบ BP                1.Rattler
    140/80 mmHg, SEM gr II LUPSB, pulse แขนแรงกว่าขา ผู ้ป่ วยมี                2.High chair
    congenital anomaly ใด                                                       3. รถเข็น
                                                                                4. รถหัดเดิน
    a. Coarctation of Aorta b. ASD            c. PDA            d. TGA          5.จุกนมหลอก

   (Ped-GI)เด็ก 9 เดือน มาด ้วย watery diarrhea ตรวจร่างกายพบ dry             (PED-Prevention)เด็กอายุ 5 ปี ตกบันได ผู ้ปกครองพามาโรงพยาบาล
    lips ทา stool exam : yellowish mucous stool เจอ WBC 5-10 จะส่ง              ตรวจร่างกายพบรอยฟกช้า ตามตัวซึงมีทังรอยเก่าและรอยใหม่ จะทา
                                                                                                                      ่    ้
    investigationใดเพือdiagnosis
                      ่                                                         อย่างไรต่อกับผู ้ป่ วยเด็กรายนี้ (ตอบข ้อ 3)
    A. Stool culture                                                            A. ให ้กลับบ ้าน
    B. Stool reducing sugar                                                     B. ปรึกษาสังคมสงเคราะห์
    C. Stool PCR for Rota virus                                                 C. รับไว ้ในโรงพยาบาล
    D. Stool occult blood
                                                                                                                                           ่
                                                                                (PED-child abuse) เด็กชายอายุ 8 ขวบ มีประวัตตกบันไดมา 1 ชัวโมง
                                                                                                                             ิ
   (Ped-Pulmo)เด็ก 5 ขวบมี sudden onset of dyspnea ไอ ฟั งปอดพบ                ก่อนมา รพ. ตรวจร่างกายพบ รอบก ้นมีผน ตามร่างกายมีรอยฟกช้าทังเก่า
                                                                                                                      ื่                      ้
    decreased breath sound ข ้างขวา จะทาอะไรต่อไป                               ใหม่กระจายทั่วร่างกาย น้ าหนักตากว่าเกณฑ์ พัฒนาการตามวัย ท่ านจะให ้
                                                                                                               ่
     a. rigid bronchoscope                                                      การักษาอย่างไร
     b. CXR
     c. fiberoptic bronchoscope                                                 ก.   admit
     d. thoracotomy
     e. ICD                                                                     ข.   ส่งปรึกษานักโภชนาการ
                                                                                ค.   ส่งปรึกษานักสังคมสงเคราะห์
                                                                                ง.   ให ้ยาแล ้วให ้กลับบ ้าน

                                                                                                                                                          11
   (PED-Endocrine)เด็กหญิง 5 ปี polydipsia polyuria น ้าหนักลด 5 kg ใน 2 เดือน                      o   muscle fiber
                                                                                                     o   peripheral nerve fiber
    มาที่ ER ด้ วย agitation, dehydration and kussmual breathing investigation:                      o   anterior horn cell
    blood sugar 520, Na 126, K 6.5, CO2 8mmol/l, urine ketone 4+
        ถาม Initial management                                                                 (PED)A 10 yr olds with uncontrol asthma present with difficult breathing
                  a)    NSS                                                                     , chest pain after several cought , PE: expiratory wheezing , diminish
                  b)    3% NaCl                                                                 breath sound left lung, heart sound to right side Dx
                  c)    7.5% NaHCO3                                                             1. Atelectasis
                  d)    IV insulin                                                              2. pleural effusion
                  e)    rectal Kayexalate                                                       3. pneumothorax
                                                                                                4. PE
                                                                                                5. acute bronchospasm
   (PED-Child dev.)เด็ก อายุ 12 เดือน ผ่าตัด cleft lip แล้ ว เหลือ cleft palate ที่ soft
    palate พ่อแม่กงวล กลัวว่าการผ่าตัดจะเป็ นอันตราย ท่านเป็ นแพทย์จะบอกพ่อแม่ว่า
                  ั                                                                         OB-GYN
                                                                                               (OB-GYN) 30 years-old G3P2 GA 38 wks. Presented with Vaginal
    ถ้ าไม่ผ่าตัดจะส่งผลเสียอย่างไรมากที่สด
                                          ุ                                                     bleeding & Abdominal pain. She has unremarkable ANC history.
                                                                                                PE: V/S – stable, fundal height 40cm above umbilicus,
        o    เสียงแหบ                                                                           Longtitudinal lie, Cephalic Presentation, Contraction every 2 mins
        o    เสียงแหลม                                                                          50sec in duration,
                                                                                                 Fetal HR 140/min reactive.
        o    ติดอ่าง                                                                             U/S- Retroplacental 4x3 hematoma at fundus
                                                                                                PV- Cx dilatation 8cm, effacement 80%, Intact membrane, station
        o    พูดเสียงขึ ้นจมูก                                                                  +2
        o    พัฒนาการด้ านการพูดช้ า                                                            What is the most appropriate Mx?
                                                                                                1. Continue spontaneous delivery
   (PED)เด็กชาย 10 ปี มีอาการอ่อนแรงขา 2 ข ้างมา 3 วัน เมือ 1 สัปดาห์
                                                               ่                                2. IV Oxytocin
    ก่อนมีไข ้ น้ ามูกไหล PE: bladder distention,muscle power grade 0 ,                         3. Amniotomy
    ชาตังแต่ใต ้สะดือลงมา, BBK:positive both side,deep tendon reflex:
        ้                                                                                       4. V/E
    absent         Which of the following is most likely diagnosis?                             5. C/S
    A. Poliomyelitis                                                                           (OB-GYN)หญิงอายุ 50 ปี 1 wk PTA ไปทา F&C มาเนื่องจากมี
    B. Transverse myelitis                                                                      abnormal vaginal bleeding มา 4 wk PTA หลังทามีไข ้ ปวดท ้องที่
    C. Viral myositis                                                                           suprapubic area ตรวจภายใน พบ foul smell and tender at
    D. Collagen vascular disease                                                                suprapubic area เกิดจากอะไร
    E. GBS
                                                                                                1.   salphingitis
   (PED)เด็กอายุ 7 ปี เริ่มเดินลาบากมา 4 ปี slow verbal , wadding gait, cranial                2.   endometritis
    nerve intact, slow reflex ถามว่าผิดปกติที่ไหน                                               3.   TOA
                                                                                                4.   reetained piece of endometrium

                                                                                                                                                                     12
   (OB-GYN) หญิงไทย 39ปี มีตกขาวสีเหลืองมา 3-4เดือน PV; yellowish                   d) F/E
    discharge with inflammed cervix with erosion ถามเกิดจากสาเหตุใด                  e) C/S
    1.HSV
    2.HPV                                                                           ผู ้ป่ วยหญิงอายุ38ปี เป็ นเบาหวานตังครรภ์ GA 30wks. มาด ้วยเจ็บครรภ์
                                                                                                                         ้
    3.N.gonorrhea                                                                    คลอดและปั สสาวะบ่อย ตรวจพบมดลูกอยู่ระดับลินปี่ คลาส่วนของทารกไม่
                                                                                                                                     ้
    4.C.trachomatis                                                                  ชัด FHS 152/min Uterine contraction 1ครังใน30นาที PV: cervix
                                                                                                                                  ้
    5.Gardnerella vaginalis                                                          dilated 1cm , effacement 50% อยากทราบว่าสาเหตุททาให ้เกิด
                                                                                                                                           ี่
                                                                                     ลักษณะในผู ้ป่ วยรายนี้คอ  ื
   (OB-GYN)หญิงหลังคลอดบุตร ปวดเต ้านมซ ้าย มีบวมแดงร ้อน ตรวจ
    ร่างกายพบ erythematous                                                           A.   Macrosomia
    crack skin around nipple and areola. จะให ้ antibiotics อะไร                     B.   Twin pregnancy
    A. Cloxacillin                                                                   C.   Myoma uteri
    B. Erythromycin                                                                  D.   Polyhydramnios
    C. Ceftriaxone                                                                   E.   Urinary tract infection

   (OB-GYN)หญิงไทยอายุ 28 ปี G1P0 GA 38 wk มาด ้วย labor pain จึง                  (OB-GYN) Pregnancy with labor pain, transverse lie ทาไรอะต่อ
    drip oxytocin ปวดที่ suprapubic area BP 90/40 mmHg PR 110 bpm                         a. Observe
    มี bleeding per vagina                                                                b. C/S
       a. sedate
       b. หยุดให ้ oxytocin                                                         (OB-GYN) หญิง 25 ปี G1P0 GA 12 wk มีอาการคลืนไส ้อาเจียนมากมา
                                                                                                                                   ่
       c. เพิม rate oxytocin
             ่
                                                                                     1 สัปดาห์ ตรวจ uterus คลา fundal height 2/3 > suprapubic จะให ้
       d. เปลียน position
               ่
       e. explore laparotomy                                                         การรักษาอะไรก่อน
                                                                                                     A. IV fluid
                                 ่ ่
    (OB-GYN)หญิง 45ปี มีก ้อนตุงทีชองคลอด ตรวจร่างกาย พบ anterior
    vaginal wall ลงมาที่ hymen, cervix เลย hymen มา 2 cm จะตรวจอะไร                  (OB-GYN) หญิงอายุ 30 ปี G3P1A1 GA 38wk มีเลือดออกทางช่อง
    เพิมเติม (ข ้อสอบเก่า)
       ่                                                                              คลอดปริมาณมาก ท ้องแข็งเกร็งเป็ นพักๆทุก 5 นาที ลูกชายอายุ 5 ปี ผ่า
    1.whiff test                                                                      คลอด มีประวัตขดมดลูก 3 ปี ก่อน PR100 BP 110/70 FH=3/4 เหนือ
                                                                                                   ิ ู
    2. fern test                                                                      สะดือ ballotment at fundus FHS 140 เป็ นอะไร
    3. Q- tip test                                                                   a. Vasa previa
    4. Pap test                                                                      b. Placenta previa
    5. Stress test                                                                   c. Abruptio placenta
                                                                                     d. Uterine rupture
   (OB-GYN)หญิงอายุ 25 ปี G3P2 GA 30 weeks มาด ้วยเจ็บท ้องคลอด                     e. Massive bloody show
    และมีน้ าเดิน PV: Cervix fully dilated, ถุงน้ าคร่าแตกแล ้ว, เด็กอยู่ในท่า
    mentoposterior, station 0, FHR 100 bpm, regular ถามว่าจะทาอะไร?                 (OB-Gyn)ผู ้หญิงอายุ 28 ปี อายุครรภ์ 28 สัปดาห์ ตรวจ VDRL reactive
    a) Spontaneous vaginal delivery                                                  titer = 1:32, FTA-ABS positive ให ้การรักษาเป็ น Benzathine
    b) Augmentation of labor                                                         penicillin 2.4 ล ้านหน่วย ทางกล ้ามเนื้อเป็ นเวลา 3 สัปดาห์ ต่อมาตรวจ
    c) V/E                                                                           ซ้าตอน อายุครรภ์ 32 สัปดาห์ VDRL = 1:8 ท่านจะให ้การรักษาที่
                                                                                     เหมาะสมทีสดอย่างไรต่อไป
                                                                                                 ่ ุ

                                                                                                                                                         13
้
    A. วัด VDRL ซา ตอนหลังคลอด 6 สัปดาห์                                                  d. Twisted ovarian
    B. ฉีด Benzathine penicillin ซ้าในขนาดและเวลาเท่าเดิม                                 e. Rupture ectopic pregnancy
    C. ฉีด Benzathine penicillin 2.4 หน่วย ครังเดียว
                                              ้
    D. ให ้penicillin หยดทางหลอดเลือดดา 1.2 ล ้านหน่วย ระยะเวลา____                 (OB-GYN)หญิง 25 ปี G1P0 GA 35 wk ประสบอุบติเหตุรถชน มาที่ ER V/S
                                                                                                                              ั
    E. เปลียนยาเป็ น erythromycin วันละ 2 กรัม 30 วัน
            ่
                                                                                     ปกติ ตรวจพบมดลูกแข็งตัวตลอดเวลา    Cervix dilate 5 cm, FHS 140 bpm
                                               ่ ่
 (OB-Gyn)ผู ้ป่ วยหญิง 45 ปี มาด ้วยก ้อนตุงทีชองคลอด 1 ปี ตรวจร่างกาย
  พบผนังช่องคลอดด ้านหน ้าหย่อนมาที่ hymen ปากมดลูกยืนเลย hymen
                                                          ่
                                                                                     ตรวจเพิ่มเติมแล้ ววินิจฉัยเป็ น Abruptio placenta จะ management อย่างไร
  มา 2 cm จะต ้องตรวจอะไรต่อ
   A. Fern test                                                                                A. Observe uterine contraction
   B. Pap test                                                                                 B. Tocolytic drug
   C. Q-tip test                                                                               C. Dexamethasone
   D. Stress test                                                                              D. Oxytocin
   E. Whiff test
                                                                                               E. C/S
 (OB-Gyn)หญิงไทย 30 ปี G2P1 เคยตังครรภ์ครังที่ 1 แท ้งตอน 6 wk
                                     ้        ้
  ครรภ์ 2 v/s Normal ทา Transvaginal U/s พบ sac แต่ไม่พบตัวเด็ก                     (OB-GYN)หญิงไทยตั ้งครรภ์ อาชีพกรรมกร อายุ 35 ปี มาด้ วยเรื่องน ้าเดิน 6
  มี bleeding per vagina ถามว่าสาเหตุน่าจะเกิดจากออะไร                               ชัวโมงก่อนมารพ คลอดด้ วยวิธี C/Sเนื่องจาก fetal distress ระหว่างรอ
                                                                                        ่
   A. Uterine abnormal
   B. Cervical incompetence                                                          คลอดถูกตรวจภายใน 4 ครั ้งใน 6 ชัวโมง 3 วันต่อมาผู้ป่วยมีไข้ Hct 24%
                                                                                                                        ่
   C. Ectropic prewgnancy                                                            ข้ อใดเป็ นปั จจัยเสียงของภาวะแทรกซ้ อ นข้ างต้ น
                                                                                                          ่
                                                                                     a. อาชีพ
    (OB-Gyn) หญิงอายุ 20 ปี G1 เจ็บท ้องคลอดมา stage 1ปกติ ผ่าน
     stage 2 ไปแล ้ว มดลูกหดตัวดี, FHS ดี, แรงเบ่งดี, PVมดลูกเปิ ด 8 cm,            b. C/S
     effacement 100 %, station +2, ROT, แต่คลาไม่ได ้ molding &                      c. PROM
     caput ภาวะทีวนจฉั ยในผู ้ป่ วยรายนี้คอ
                 ่ ิ ิ                    ื                                          d. anemia
    A. mentoposterior                                                                e. PV 4 ครั ้ง
    B. persistent occiput posterior
    C. deep arrest transverse                                                     (OB-GYN) หญิง 25 ปี G1P0 GA 38wk มีอาการปวดศีรษะ รู้สกว่าลูกดิ ้นน้ อยลง
                                                                                                                                        ึ
    D. hypotonic uterine contraction
    E. Cephalopelvic disproportion                                                 PE BP 180/110mmHg T 37C RR 20/min uterine size = GA proteinuria 2+
                                                                                   DTR 3+ ควรให้ ยาใด
 (OB-GYN) ผู้หญิงไทยอายุ 25ปี มีไข้ สง ปวดท้ องมา 10วัน ตรวจร่างกายพบ
                                      ู
                                                                                          1.   Diazepam
     abdominal mass, peritonitis, no guarding, cervix excitation pain positive
                                                                                          2.   Magnesium sulphate
     What is most likely diagnosis?                                                       3.   Antihypertensive drug
             a. Tubo-ovarian abscess                                                      4.   ……..
             b. PID                                                                       5.   ………..
             c. Appendiceal abscess

                                                                                                                                                            14
 (OB-GYN)G1P1 GA 38 wks. มาด้ วยน ้าเดินมา5วัน คลอดปกติ หลังจาก                 (Surg-Uro)ชาย 45 ปี 2 ปี ก่อน ตรวจพบ Rt. ureteric stone size 1.5 cm
                                                                                  ผู ้ป่ วยปฏิเสธการรักษา 2 วันก่อน ปั สสาวะไม่ออก หอบเหนื่อย U/S พบ
  คลอด1วันมีไข้ T.39.5 พูดจาสับสน จาสามีไม่ได้ เห็นผีบอกว่าอย่ามาหลอกกันเลย       Rt. severe hydronephrosis, Lt. moderate hydronephrosis with Lt.
  จงวินิจฉัย                                                                      renal stone 2 cm LAB: Na 130, K 7, Cl 100, CO2 12, Cr 9.8 What is
                                                                                  the most appropriate management?
            1.   Dilirium                                                         a. ESWL Lt. renal stone
            2.   D. Postpartum psychosis                                          b. ESWL Rt. Ureteric stone
            3.   Brief psychosis                                                  c. Lt. nephrolithotomy
            4.   Postpartum depression                                            d. Lt. percutaneous nephrotomy
            5.   Postpartum blue                                                  e. Rt. transurethral laser lithotripsy

                                                                                 (Sx URO) ชายอายุ 65 ปี เป็ นเบาหวานมา 10 ปี 3 เดือนก่อนมาพบ
                                                                                  แพทย์ด ้วยเรืองปั สสาวะบ่อย ปวดเบ่ง ปั สสาวะไม่พุ่ง ได ้ α-blocker
                                                                                                 ่
Surgery                                                                           อาการไม่ดขน วันนี้มาด ้วยปั สสาวะไม่ออกมา 1 วัน PRmild enlarge
                                                                                            ี ึ้
   (Surg)ชาย 40 เป็ นก ้อนทีหลัง 10 เซนติเมตร บริเวณ lumbrosacral area
                              ่                                                   prostate gland, smooth surface, rubbery และมี loose sphincter
    ก ้อนไม่ตด skin ไม่เจ็บ ไม่มรอยบุ๋ม
             ิ                  ี                                                 tone แพทย์พจารณาผ่าตัด TUR-P จะส่งตรวจอะไรเพิมเติมทีสาคัญทีสด
                                                                                                   ิ                                  ่        ่     ่ ุ
    A. Lipoma                                                                     เพือพิจารณาทา TUR-P
                                                                                     ่
    B. Meningioma                                                                 A. Plain KUB
    C. Chordoma                                                                   B. cystrometry
    D. Epidermal cyst                                                             C. Urethrography
                                                                                  D. panoscopy
   (Surg)ชายไทยอายุ 50 ปี มีก ้อนโตทีหลังมา 3 ปี ค่อยๆโต ไม่เจ็บ
                                      ่                                           E. transrectal U/S
    movable , soft sonsistency จงให ้การวินจฉั ย
                                           ิ
     a. lipoma
     b. epidermal cyst
                                                                               (Surg) ผู้ป่วยชาย 18 ปี ปวดท้ องรอบสะดือมา 1 วัน 2 hr ก่อนมา รพ. ปวดมากขึ ้น
     c. melanoma                                                                ย้ ายมาปวดที่ RLQ ไม่มีคลืนไส้ อาเจียน มีเบื่ออาหาร ตรวจร่างกายพบ T 37.0c,
                                                                                                          ่
     d. meningocele
     e. teratoma
                                                                                tender with guarding at RLQ, no rebound, diminished bowel sound, Lab มี
                                                                                WBC 3,000 ( N 60%, L 40% ) ส่ง investigation อะไรในขณะนี ้
   (SURGERY) ผู ้ป่ วยหญิงอายุ 40 ปี ผ่าตัดคลอดลูก 2 ครัง มีปวดท ้อง
                                                         ้
    บริเวณตรงกลางเป็ นๆหายๆ ไม่ถ่าย ไม่ผายลมมา 1 วัน PE T 38 C BP               A. UA
    130/80 mmHg P 100/min RR 18/min มีปากแห ้ง ท ้องอืด bowel                   B. CT lower abdomen
    sound ลดลง มี mass ที่ Right lower quadrant abdomen ขอบเขตคลา
            ั
    ได ้ไม่ชดเจน Guarding at RLQ CBC Hct 40% WBC 15,000(N 80% L
                                                                                C. diagnostic laparoscopy
    20%) จะส่ง Investigation อะไรเป็ นอันดับแรกเพือการวินจฉั ย
                                                   ่       ิ                    D. ไม่ต้องทาอะไร รอ 24 hr
    A.   Plain film abdomen                                                     E. ไม่ต้องทาอะไร ผ่าตัดเลย
    B.   U/S lower abdomen
    C.   CT scan whole abdomen
    D.   Barium enema
    E.   Colonoscopy

                                                                                                                                                               15
Compre Rama 2010
Compre Rama 2010
Compre Rama 2010
Compre Rama 2010
Compre Rama 2010
Compre Rama 2010
Compre Rama 2010
Compre Rama 2010

More Related Content

What's hot

Step3 Tutorial by SWU book1
Step3 Tutorial by SWU book1Step3 Tutorial by SWU book1
Step3 Tutorial by SWU book1vora kun
 
5.แนวข้อสอบ o net สุขศึกษา(ม.3)
5.แนวข้อสอบ o net สุขศึกษา(ม.3)5.แนวข้อสอบ o net สุขศึกษา(ม.3)
5.แนวข้อสอบ o net สุขศึกษา(ม.3)teerachon
 
ประวัติลูกเสือโลก
ประวัติลูกเสือโลกประวัติลูกเสือโลก
ประวัติลูกเสือโลกNew Nan
 
Cpg osteoarthritis or osteoarthrosis 2554
Cpg osteoarthritis or osteoarthrosis 2554Cpg osteoarthritis or osteoarthrosis 2554
Cpg osteoarthritis or osteoarthrosis 2554Utai Sukviwatsirikul
 
สุขฯ ม.2 หน่วย 8
สุขฯ ม.2 หน่วย 8สุขฯ ม.2 หน่วย 8
สุขฯ ม.2 หน่วย 8supap6259
 
การจัดการความปวดเบื้องต้น
การจัดการความปวดเบื้องต้นการจัดการความปวดเบื้องต้น
การจัดการความปวดเบื้องต้นSutthiluck Kaewboonrurn
 
Clinical practice guidelines for hemorrhagic stroke
Clinical practice guidelines for hemorrhagic strokeClinical practice guidelines for hemorrhagic stroke
Clinical practice guidelines for hemorrhagic strokeUtai Sukviwatsirikul
 
ตรวจร่างกายระบบประสาท
ตรวจร่างกายระบบประสาทตรวจร่างกายระบบประสาท
ตรวจร่างกายระบบประสาทCotton On
 
แผ่นพับ ข้อเข่าเสื่อม
แผ่นพับ ข้อเข่าเสื่อมแผ่นพับ ข้อเข่าเสื่อม
แผ่นพับ ข้อเข่าเสื่อมNattha Namm
 
การออกกำลังกายในผู้ป่วยโรคไตเรื้อรัง
การออกกำลังกายในผู้ป่วยโรคไตเรื้อรังการออกกำลังกายในผู้ป่วยโรคไตเรื้อรัง
การออกกำลังกายในผู้ป่วยโรคไตเรื้อรังCAPD AngThong
 
สุขฯ ม.2 หน่วย 3
สุขฯ ม.2 หน่วย 3สุขฯ ม.2 หน่วย 3
สุขฯ ม.2 หน่วย 3supap6259
 
11แผน
11แผน11แผน
11แผนFmz Npaz
 
ไม่ล้มไม่ลืม
ไม่ล้มไม่ลืมไม่ล้มไม่ลืม
ไม่ล้มไม่ลืมUtai Sukviwatsirikul
 
การพยาบาลผู้ป่วยผู้ใหญ่ที่มีปัญหาเกี่ยวกับระบบสืบพันธุ์และนรีเวชวิทยา
การพยาบาลผู้ป่วยผู้ใหญ่ที่มีปัญหาเกี่ยวกับระบบสืบพันธุ์และนรีเวชวิทยาการพยาบาลผู้ป่วยผู้ใหญ่ที่มีปัญหาเกี่ยวกับระบบสืบพันธุ์และนรีเวชวิทยา
การพยาบาลผู้ป่วยผู้ใหญ่ที่มีปัญหาเกี่ยวกับระบบสืบพันธุ์และนรีเวชวิทยาChutchavarn Wongsaree
 
การดูแลความต้องการพื้นฐานของบุคคลด้านการทรงตัว การเคลื่อนไหว การจำกัดการเคลื่...
การดูแลความต้องการพื้นฐานของบุคคลด้านการทรงตัว การเคลื่อนไหว การจำกัดการเคลื่...การดูแลความต้องการพื้นฐานของบุคคลด้านการทรงตัว การเคลื่อนไหว การจำกัดการเคลื่...
การดูแลความต้องการพื้นฐานของบุคคลด้านการทรงตัว การเคลื่อนไหว การจำกัดการเคลื่...CC Nakhon Pathom Rajabhat University
 
RCA แบบ ชิว ชิว
RCA แบบ ชิว ชิวRCA แบบ ชิว ชิว
RCA แบบ ชิว ชิวSuradet Sriangkoon
 

What's hot (20)

Step3 Tutorial by SWU book1
Step3 Tutorial by SWU book1Step3 Tutorial by SWU book1
Step3 Tutorial by SWU book1
 
5.แนวข้อสอบ o net สุขศึกษา(ม.3)
5.แนวข้อสอบ o net สุขศึกษา(ม.3)5.แนวข้อสอบ o net สุขศึกษา(ม.3)
5.แนวข้อสอบ o net สุขศึกษา(ม.3)
 
ประวัติลูกเสือโลก
ประวัติลูกเสือโลกประวัติลูกเสือโลก
ประวัติลูกเสือโลก
 
22
2222
22
 
Diabetic ketoacidosis
Diabetic ketoacidosisDiabetic ketoacidosis
Diabetic ketoacidosis
 
Cpg osteoarthritis or osteoarthrosis 2554
Cpg osteoarthritis or osteoarthrosis 2554Cpg osteoarthritis or osteoarthrosis 2554
Cpg osteoarthritis or osteoarthrosis 2554
 
สุขฯ ม.2 หน่วย 8
สุขฯ ม.2 หน่วย 8สุขฯ ม.2 หน่วย 8
สุขฯ ม.2 หน่วย 8
 
การจัดการความปวดเบื้องต้น
การจัดการความปวดเบื้องต้นการจัดการความปวดเบื้องต้น
การจัดการความปวดเบื้องต้น
 
Clinical practice guidelines for hemorrhagic stroke
Clinical practice guidelines for hemorrhagic strokeClinical practice guidelines for hemorrhagic stroke
Clinical practice guidelines for hemorrhagic stroke
 
ตรวจร่างกายระบบประสาท
ตรวจร่างกายระบบประสาทตรวจร่างกายระบบประสาท
ตรวจร่างกายระบบประสาท
 
แผ่นพับ ข้อเข่าเสื่อม
แผ่นพับ ข้อเข่าเสื่อมแผ่นพับ ข้อเข่าเสื่อม
แผ่นพับ ข้อเข่าเสื่อม
 
การเลี้ยงไก่พื้นเมือง
การเลี้ยงไก่พื้นเมืองการเลี้ยงไก่พื้นเมือง
การเลี้ยงไก่พื้นเมือง
 
การออกกำลังกายในผู้ป่วยโรคไตเรื้อรัง
การออกกำลังกายในผู้ป่วยโรคไตเรื้อรังการออกกำลังกายในผู้ป่วยโรคไตเรื้อรัง
การออกกำลังกายในผู้ป่วยโรคไตเรื้อรัง
 
สุขฯ ม.2 หน่วย 3
สุขฯ ม.2 หน่วย 3สุขฯ ม.2 หน่วย 3
สุขฯ ม.2 หน่วย 3
 
11แผน
11แผน11แผน
11แผน
 
คู่มือโภชนากร
คู่มือโภชนากรคู่มือโภชนากร
คู่มือโภชนากร
 
ไม่ล้มไม่ลืม
ไม่ล้มไม่ลืมไม่ล้มไม่ลืม
ไม่ล้มไม่ลืม
 
การพยาบาลผู้ป่วยผู้ใหญ่ที่มีปัญหาเกี่ยวกับระบบสืบพันธุ์และนรีเวชวิทยา
การพยาบาลผู้ป่วยผู้ใหญ่ที่มีปัญหาเกี่ยวกับระบบสืบพันธุ์และนรีเวชวิทยาการพยาบาลผู้ป่วยผู้ใหญ่ที่มีปัญหาเกี่ยวกับระบบสืบพันธุ์และนรีเวชวิทยา
การพยาบาลผู้ป่วยผู้ใหญ่ที่มีปัญหาเกี่ยวกับระบบสืบพันธุ์และนรีเวชวิทยา
 
การดูแลความต้องการพื้นฐานของบุคคลด้านการทรงตัว การเคลื่อนไหว การจำกัดการเคลื่...
การดูแลความต้องการพื้นฐานของบุคคลด้านการทรงตัว การเคลื่อนไหว การจำกัดการเคลื่...การดูแลความต้องการพื้นฐานของบุคคลด้านการทรงตัว การเคลื่อนไหว การจำกัดการเคลื่...
การดูแลความต้องการพื้นฐานของบุคคลด้านการทรงตัว การเคลื่อนไหว การจำกัดการเคลื่...
 
RCA แบบ ชิว ชิว
RCA แบบ ชิว ชิวRCA แบบ ชิว ชิว
RCA แบบ ชิว ชิว
 

Viewers also liked

Compre si 2010 l
Compre si 2010 lCompre si 2010 l
Compre si 2010 lvora kun
 
Mdcu Step2 Gen Sx Ii
Mdcu Step2 Gen Sx IiMdcu Step2 Gen Sx Ii
Mdcu Step2 Gen Sx Iivora kun
 
ศรว 51 ANS By Cmu
ศรว 51 ANS By Cmuศรว 51 ANS By Cmu
ศรว 51 ANS By Cmuvora kun
 
Nt2009 complete all
Nt2009 complete allNt2009 complete all
Nt2009 complete allvora kun
 
Survivor NT step2 SIRIRAJ book 1
Survivor NT step2 SIRIRAJ book 1Survivor NT step2 SIRIRAJ book 1
Survivor NT step2 SIRIRAJ book 1vora kun
 
Newborn nt ปี 5
Newborn nt ปี 5Newborn nt ปี 5
Newborn nt ปี 5Hummd Mdhum
 
Step3 Tutorial by SWU book2
Step3 Tutorial by SWU book2Step3 Tutorial by SWU book2
Step3 Tutorial by SWU book2vora kun
 
Clinical medicine
Clinical medicineClinical medicine
Clinical medicineMoni Buvy
 
Osce ศรว ครั้งแรก dec52
Osce ศรว ครั้งแรก dec52Osce ศรว ครั้งแรก dec52
Osce ศรว ครั้งแรก dec52vora kun
 
Survival for all draft 1 - 3
Survival for all draft 1  - 3Survival for all draft 1  - 3
Survival for all draft 1 - 3Domo Kwan
 
ข้อสอบกายวิภาคศาสตร์ (บันทึกอัตโนมัติ)
ข้อสอบกายวิภาคศาสตร์ (บันทึกอัตโนมัติ)ข้อสอบกายวิภาคศาสตร์ (บันทึกอัตโนมัติ)
ข้อสอบกายวิภาคศาสตร์ (บันทึกอัตโนมัติ)Aom S
 
การประดับธงชาติ กลุ่ม 3
การประดับธงชาติ กลุ่ม 3การประดับธงชาติ กลุ่ม 3
การประดับธงชาติ กลุ่ม 3PamPaul
 
การเจริญเติบโตของเอ็มบริโอในครรภ์ 29,30
การเจริญเติบโตของเอ็มบริโอในครรภ์ 29,30การเจริญเติบโตของเอ็มบริโอในครรภ์ 29,30
การเจริญเติบโตของเอ็มบริโอในครรภ์ 29,30PamPaul
 
NT step2 march 53
NT step2 march 53NT step2 march 53
NT step2 march 53vora kun
 
ortho 02 orthopaedic complication & prevention + orthopaedic trauma (practica...
ortho 02 orthopaedic complication & prevention + orthopaedic trauma (practica...ortho 02 orthopaedic complication & prevention + orthopaedic trauma (practica...
ortho 02 orthopaedic complication & prevention + orthopaedic trauma (practica...vora kun
 
Approach & evaluation of patient with somatic pain
Approach & evaluation of patient with somatic painApproach & evaluation of patient with somatic pain
Approach & evaluation of patient with somatic painDr. Md. Rashedul Islam
 
CPR 2010 อ ปริญญา รามา
CPR 2010 อ ปริญญา รามาCPR 2010 อ ปริญญา รามา
CPR 2010 อ ปริญญา รามาvora kun
 

Viewers also liked (20)

รวมข้อสอบCompre nl
รวมข้อสอบCompre nlรวมข้อสอบCompre nl
รวมข้อสอบCompre nl
 
Compre si 2010 l
Compre si 2010 lCompre si 2010 l
Compre si 2010 l
 
National test _2553_TU
National test _2553_TUNational test _2553_TU
National test _2553_TU
 
Mdcu Step2 Gen Sx Ii
Mdcu Step2 Gen Sx IiMdcu Step2 Gen Sx Ii
Mdcu Step2 Gen Sx Ii
 
ศรว 51 ANS By Cmu
ศรว 51 ANS By Cmuศรว 51 ANS By Cmu
ศรว 51 ANS By Cmu
 
Nt2009 complete all
Nt2009 complete allNt2009 complete all
Nt2009 complete all
 
Survivor NT step2 SIRIRAJ book 1
Survivor NT step2 SIRIRAJ book 1Survivor NT step2 SIRIRAJ book 1
Survivor NT step2 SIRIRAJ book 1
 
Newborn nt ปี 5
Newborn nt ปี 5Newborn nt ปี 5
Newborn nt ปี 5
 
Step3 Tutorial by SWU book2
Step3 Tutorial by SWU book2Step3 Tutorial by SWU book2
Step3 Tutorial by SWU book2
 
Clinical medicine
Clinical medicineClinical medicine
Clinical medicine
 
Osce ศรว ครั้งแรก dec52
Osce ศรว ครั้งแรก dec52Osce ศรว ครั้งแรก dec52
Osce ศรว ครั้งแรก dec52
 
Survival for all draft 1 - 3
Survival for all draft 1  - 3Survival for all draft 1  - 3
Survival for all draft 1 - 3
 
ข้อสอบกายวิภาคศาสตร์ (บันทึกอัตโนมัติ)
ข้อสอบกายวิภาคศาสตร์ (บันทึกอัตโนมัติ)ข้อสอบกายวิภาคศาสตร์ (บันทึกอัตโนมัติ)
ข้อสอบกายวิภาคศาสตร์ (บันทึกอัตโนมัติ)
 
การประดับธงชาติ กลุ่ม 3
การประดับธงชาติ กลุ่ม 3การประดับธงชาติ กลุ่ม 3
การประดับธงชาติ กลุ่ม 3
 
การเจริญเติบโตของเอ็มบริโอในครรภ์ 29,30
การเจริญเติบโตของเอ็มบริโอในครรภ์ 29,30การเจริญเติบโตของเอ็มบริโอในครรภ์ 29,30
การเจริญเติบโตของเอ็มบริโอในครรภ์ 29,30
 
Swu
SwuSwu
Swu
 
NT step2 march 53
NT step2 march 53NT step2 march 53
NT step2 march 53
 
ortho 02 orthopaedic complication & prevention + orthopaedic trauma (practica...
ortho 02 orthopaedic complication & prevention + orthopaedic trauma (practica...ortho 02 orthopaedic complication & prevention + orthopaedic trauma (practica...
ortho 02 orthopaedic complication & prevention + orthopaedic trauma (practica...
 
Approach & evaluation of patient with somatic pain
Approach & evaluation of patient with somatic painApproach & evaluation of patient with somatic pain
Approach & evaluation of patient with somatic pain
 
CPR 2010 อ ปริญญา รามา
CPR 2010 อ ปริญญา รามาCPR 2010 อ ปริญญา รามา
CPR 2010 อ ปริญญา รามา
 

Similar to Compre Rama 2010

2007821172158 466 6438_1
2007821172158 466 6438_12007821172158 466 6438_1
2007821172158 466 6438_1New Srsn
 
สอบ-ศรว-มีนาคม-2551
สอบ-ศรว-มีนาคม-2551สอบ-ศรว-มีนาคม-2551
สอบ-ศรว-มีนาคม-2551rookiess
 
Nle step 2_2009 si115-116 and nle_step_2_2009 nctms editors cut
Nle step 2_2009 si115-116 and nle_step_2_2009 nctms editors cutNle step 2_2009 si115-116 and nle_step_2_2009 nctms editors cut
Nle step 2_2009 si115-116 and nle_step_2_2009 nctms editors cutLoveis1able Khumpuangdee
 
Exercise national license_part_ii_march_2009_2
Exercise national license_part_ii_march_2009_2Exercise national license_part_ii_march_2009_2
Exercise national license_part_ii_march_2009_2Loveis1able Khumpuangdee
 
Nle step 2_2009 si115-116 and nle_step_2_2009 nctms editors cut key
Nle step 2_2009 si115-116 and nle_step_2_2009 nctms editors cut keyNle step 2_2009 si115-116 and nle_step_2_2009 nctms editors cut key
Nle step 2_2009 si115-116 and nle_step_2_2009 nctms editors cut keyLoveis1able Khumpuangdee
 
Nt2009 Complete Ans
Nt2009 Complete AnsNt2009 Complete Ans
Nt2009 Complete Ansvora kun
 
Comprehensive 2008 kku
Comprehensive 2008 kkuComprehensive 2008 kku
Comprehensive 2008 kkurookiess
 
กาย่า presentation
กาย่า presentationกาย่า presentation
กาย่า presentationCrystalpet
 
Motor weakness and Cerebrovascular Disease
Motor weakness and Cerebrovascular DiseaseMotor weakness and Cerebrovascular Disease
Motor weakness and Cerebrovascular DiseaseNarongrit Kasemsap
 
Mdcu Exam Step 2 2010
Mdcu Exam Step 2 2010Mdcu Exam Step 2 2010
Mdcu Exam Step 2 2010vora kun
 
10แบบทดสอบภูมิคุ้มกันของร่างกาย (ตอนที่ 1)
10แบบทดสอบภูมิคุ้มกันของร่างกาย (ตอนที่ 1)10แบบทดสอบภูมิคุ้มกันของร่างกาย (ตอนที่ 1)
10แบบทดสอบภูมิคุ้มกันของร่างกาย (ตอนที่ 1)สำเร็จ นางสีคุณ
 

Similar to Compre Rama 2010 (20)

2007821172158 466 6438_1
2007821172158 466 6438_12007821172158 466 6438_1
2007821172158 466 6438_1
 
สอบ-ศรว-มีนาคม-2551
สอบ-ศรว-มีนาคม-2551สอบ-ศรว-มีนาคม-2551
สอบ-ศรว-มีนาคม-2551
 
Nle step 2_2009 si115-116 and nle_step_2_2009 nctms editors cut
Nle step 2_2009 si115-116 and nle_step_2_2009 nctms editors cutNle step 2_2009 si115-116 and nle_step_2_2009 nctms editors cut
Nle step 2_2009 si115-116 and nle_step_2_2009 nctms editors cut
 
For extern
For externFor extern
For extern
 
Exercise national license_part_ii_march_2009_2
Exercise national license_part_ii_march_2009_2Exercise national license_part_ii_march_2009_2
Exercise national license_part_ii_march_2009_2
 
Nl part ii march 2009
Nl part ii march 2009Nl part ii march 2009
Nl part ii march 2009
 
Nl part ii march 2009
Nl part ii march 2009Nl part ii march 2009
Nl part ii march 2009
 
Nle step 2_2009 si115-116 and nle_step_2_2009 nctms editors cut key
Nle step 2_2009 si115-116 and nle_step_2_2009 nctms editors cut keyNle step 2_2009 si115-116 and nle_step_2_2009 nctms editors cut key
Nle step 2_2009 si115-116 and nle_step_2_2009 nctms editors cut key
 
Nt2009 Complete Ans
Nt2009 Complete AnsNt2009 Complete Ans
Nt2009 Complete Ans
 
Comprehensive 2008 kku
Comprehensive 2008 kkuComprehensive 2008 kku
Comprehensive 2008 kku
 
Case01
Case01Case01
Case01
 
กาย่า presentation
กาย่า presentationกาย่า presentation
กาย่า presentation
 
Motor weakness and Cerebrovascular Disease
Motor weakness and Cerebrovascular DiseaseMotor weakness and Cerebrovascular Disease
Motor weakness and Cerebrovascular Disease
 
Mdcu Exam Step 2 2010
Mdcu Exam Step 2 2010Mdcu Exam Step 2 2010
Mdcu Exam Step 2 2010
 
แบบฝึกหัด
แบบฝึกหัดแบบฝึกหัด
แบบฝึกหัด
 
test
testtest
test
 
10แบบทดสอบภูมิคุ้มกันของร่างกาย (ตอนที่ 1)
10แบบทดสอบภูมิคุ้มกันของร่างกาย (ตอนที่ 1)10แบบทดสอบภูมิคุ้มกันของร่างกาย (ตอนที่ 1)
10แบบทดสอบภูมิคุ้มกันของร่างกาย (ตอนที่ 1)
 
Ihd
IhdIhd
Ihd
 
กรณีศึกษาไต (Ns) แก้ไข
กรณีศึกษาไต (Ns)  แก้ไขกรณีศึกษาไต (Ns)  แก้ไข
กรณีศึกษาไต (Ns) แก้ไข
 
National license 2010 by med tu 16
National license 2010 by med tu 16National license 2010 by med tu 16
National license 2010 by med tu 16
 

More from vora kun

ประชุมวิชาการ ศิริราช 53
ประชุมวิชาการ ศิริราช 53ประชุมวิชาการ ศิริราช 53
ประชุมวิชาการ ศิริราช 53vora kun
 
ประชุมวิชาการ ศิริราช 52
ประชุมวิชาการ ศิริราช 52ประชุมวิชาการ ศิริราช 52
ประชุมวิชาการ ศิริราช 52vora kun
 
Nt2553step3round1 28NOV2553
Nt2553step3round1 28NOV2553Nt2553step3round1 28NOV2553
Nt2553step3round1 28NOV2553vora kun
 
Osce ศรว ครั้งที่สอง 10jan53
Osce ศรว ครั้งที่สอง 10jan53Osce ศรว ครั้งที่สอง 10jan53
Osce ศรว ครั้งที่สอง 10jan53vora kun
 
Osce คณะ si 115
Osce คณะ si 115Osce คณะ si 115
Osce คณะ si 115vora kun
 
Thai Osteoporosis guideline 2553
Thai Osteoporosis guideline 2553Thai Osteoporosis guideline 2553
Thai Osteoporosis guideline 2553vora kun
 
Abnormal pap smear ศิริราช ppt
Abnormal pap smear ศิริราช pptAbnormal pap smear ศิริราช ppt
Abnormal pap smear ศิริราช pptvora kun
 
ortho 02 orthopaedic complication & prevention + orthopaedic trauma (practica...
ortho 02 orthopaedic complication & prevention + orthopaedic trauma (practica...ortho 02 orthopaedic complication & prevention + orthopaedic trauma (practica...
ortho 02 orthopaedic complication & prevention + orthopaedic trauma (practica...vora kun
 
ortho 06 common ortho dis 2 edited 12 mar 10
ortho 06 common ortho dis 2 edited 12 mar 10ortho 06 common ortho dis 2 edited 12 mar 10
ortho 06 common ortho dis 2 edited 12 mar 10vora kun
 
ortho 05 common rheumatic dx rx
ortho 05 common rheumatic dx rxortho 05 common rheumatic dx rx
ortho 05 common rheumatic dx rxvora kun
 
ortho 01 management of open fracture-update by kk 31052010
ortho 01 management of open fracture-update by kk 31052010ortho 01 management of open fracture-update by kk 31052010
ortho 01 management of open fracture-update by kk 31052010vora kun
 
ortho 04 drugs in orthopaedic (principle & common use)
ortho 04 drugs in orthopaedic (principle & common use)ortho 04 drugs in orthopaedic (principle & common use)
ortho 04 drugs in orthopaedic (principle & common use)vora kun
 
ortho 03 principle of closed reduction in fracture and dislocation
ortho 03 principle of closed reduction in fracture and dislocationortho 03 principle of closed reduction in fracture and dislocation
ortho 03 principle of closed reduction in fracture and dislocationvora kun
 
หัตถการที่จำเป็นทางสูติ
หัตถการที่จำเป็นทางสูติหัตถการที่จำเป็นทางสูติ
หัตถการที่จำเป็นทางสูติvora kun
 
SWU CXR interpretation
SWU  CXR interpretationSWU  CXR interpretation
SWU CXR interpretationvora kun
 
Total parenteral nutrition
Total parenteral nutritionTotal parenteral nutrition
Total parenteral nutritionvora kun
 
NeuroSx step2 Review
NeuroSx step2 ReviewNeuroSx step2 Review
NeuroSx step2 Reviewvora kun
 
ศรว 51 By Cmu
ศรว 51 By Cmuศรว 51 By Cmu
ศรว 51 By Cmuvora kun
 

More from vora kun (18)

ประชุมวิชาการ ศิริราช 53
ประชุมวิชาการ ศิริราช 53ประชุมวิชาการ ศิริราช 53
ประชุมวิชาการ ศิริราช 53
 
ประชุมวิชาการ ศิริราช 52
ประชุมวิชาการ ศิริราช 52ประชุมวิชาการ ศิริราช 52
ประชุมวิชาการ ศิริราช 52
 
Nt2553step3round1 28NOV2553
Nt2553step3round1 28NOV2553Nt2553step3round1 28NOV2553
Nt2553step3round1 28NOV2553
 
Osce ศรว ครั้งที่สอง 10jan53
Osce ศรว ครั้งที่สอง 10jan53Osce ศรว ครั้งที่สอง 10jan53
Osce ศรว ครั้งที่สอง 10jan53
 
Osce คณะ si 115
Osce คณะ si 115Osce คณะ si 115
Osce คณะ si 115
 
Thai Osteoporosis guideline 2553
Thai Osteoporosis guideline 2553Thai Osteoporosis guideline 2553
Thai Osteoporosis guideline 2553
 
Abnormal pap smear ศิริราช ppt
Abnormal pap smear ศิริราช pptAbnormal pap smear ศิริราช ppt
Abnormal pap smear ศิริราช ppt
 
ortho 02 orthopaedic complication & prevention + orthopaedic trauma (practica...
ortho 02 orthopaedic complication & prevention + orthopaedic trauma (practica...ortho 02 orthopaedic complication & prevention + orthopaedic trauma (practica...
ortho 02 orthopaedic complication & prevention + orthopaedic trauma (practica...
 
ortho 06 common ortho dis 2 edited 12 mar 10
ortho 06 common ortho dis 2 edited 12 mar 10ortho 06 common ortho dis 2 edited 12 mar 10
ortho 06 common ortho dis 2 edited 12 mar 10
 
ortho 05 common rheumatic dx rx
ortho 05 common rheumatic dx rxortho 05 common rheumatic dx rx
ortho 05 common rheumatic dx rx
 
ortho 01 management of open fracture-update by kk 31052010
ortho 01 management of open fracture-update by kk 31052010ortho 01 management of open fracture-update by kk 31052010
ortho 01 management of open fracture-update by kk 31052010
 
ortho 04 drugs in orthopaedic (principle & common use)
ortho 04 drugs in orthopaedic (principle & common use)ortho 04 drugs in orthopaedic (principle & common use)
ortho 04 drugs in orthopaedic (principle & common use)
 
ortho 03 principle of closed reduction in fracture and dislocation
ortho 03 principle of closed reduction in fracture and dislocationortho 03 principle of closed reduction in fracture and dislocation
ortho 03 principle of closed reduction in fracture and dislocation
 
หัตถการที่จำเป็นทางสูติ
หัตถการที่จำเป็นทางสูติหัตถการที่จำเป็นทางสูติ
หัตถการที่จำเป็นทางสูติ
 
SWU CXR interpretation
SWU  CXR interpretationSWU  CXR interpretation
SWU CXR interpretation
 
Total parenteral nutrition
Total parenteral nutritionTotal parenteral nutrition
Total parenteral nutrition
 
NeuroSx step2 Review
NeuroSx step2 ReviewNeuroSx step2 Review
NeuroSx step2 Review
 
ศรว 51 By Cmu
ศรว 51 By Cmuศรว 51 By Cmu
ศรว 51 By Cmu
 

Compre Rama 2010

  • 1. COMPREHENSIVE “พยายามรวมเท่าที่ทาได้ อะ… จริ งๆ ลองดูของปี 2008ด้ วยละกันนะคล้ ายกันอยู่เหมือนกัน” TEST Thanks เพื่อนๆ ที่ช่วยจาข้ อสอบออกมา ทั ้ง Rama และ PI นะค้ าบบ , เพื่อนๆ ที่ช่วยเราแยกข้ อสอบตาม ward RAMA & PI 2010 1
  • 2. MEDICINE A. add lamivudine B. add interferon-alpha  (Med-Neuro) 64 years-old Male with history of drinking 4 bottle4 C. add steroid D. หยุด diclofenac of beer for 10 years. Presents with 4 episodes of 1-2 mins of E. advice splenectomy Generalized Tonic Clonic seizure. Between each attack patient gains consciousness completely. PE:Confusion, Signs of chronic  (MED-GI) หญิงอายุ 30 ปี มาตรวจสุขภาพ ผลปรากฏว่า HBsAg + , liver disease, resting tremor without flapping tremor, Normal HBsAb -, ผล liver function test ปกติ ควรทาอย่างไรมากทีสด ่ ุ Neuro exam. What is an immediate Mx? A. รักษาด ้วย interferon B. ให ้ HBV vaccine 1. Load Oral Hydantoin C. ตรวจ U/S abdomen D. ตรวจ AFP 2. Load IV Hydantoin E. ตรวจ HBsAg ซ้าในอีก 6 เดือน 3. Load IV Benzodiazepine  (MED-GI) ชายไทย 45ปี ปวดใต ้ชายโครงขวามา 5 วัน 10วันpta มี 4. Load IV Barbidurate อาการถ่ายเป็ นมูกเลือด v/s 38 c P 80 RR 16 BP 120/80 PE. 5. Treat Hepatic encephalopathy พบคลาตับได ้ 3 cm . กดเจ็บ มี jaundice จะพิจารณาให ้ ATB อะไร  (Med-Neuro) 60 years-old woman presents with right drooping 1. Cloxacillin eyelid for 3 months. There is fluctuation of symptom. Rt. Eyelid 2. Cotrimazole almost normal in the morning. On examination – Moderate 3. Quinolone derivertive drooping Rt. Eyelid, Full EOM, Equal pupil, Normal muscles power, 4. Nitronidazole derivertive Normal DTR What is the most likely diagnosis?  (Med-ID)หญิงอายุ 45 ปี U/D เป็ น SLE on steroid มาด ้วยเริมมี ่ 1.senile ptosis ไข ้ ไอ หายใจลาบาก CXR (1 mo ago) พบ RLL mass size 1*1 2.MG cm F/U CXR (ครังนี้) พบว่า mass มีขนาดใหญ่ขน (จาขนาด ้ ึ้ 3.Horner’s syndrome ไม่ได ้) ตรวจร่างกาย มี questionable stiffness of neck LP พบ WBC 4.CNIII palsy เป็ น mononuclear cell จานวน 65, Glucose 65(blood sugar 100), 5.Oculopharyngeal muscular dystrophy Protein 45 ถามว่าเกิดจาก? a) Penicillosis  (Med-Hemato) หญิง 50 ปี ปวดสะโพกขวา 3 เดือน ตรวจร่างกายมีซด, ี b) Histoplasmosis limit ROM right pelvis due to pain, X-ray มี osteolytic lesion ที่ right c) Candidiasis pelvis involve acetabulum และมี scattered osteolytic lesion ที่ skull d) Aspergillosis PBS มีรูปให ้ดู ดูเป็ น Rouleaux formation e) Cryptococcus Diagnostic investigation ทีเหมาะทีสดคือ ่ ่ ุ 1.Serum Ca, PO4 level  (Med-ID)ผู ้ป่ วยท ้องเสียเรือรัง ตรวจอุจจาระพบตามรูป เป็ น capillaria ควร ้ 2.Protein electrophoresis ป้ องกันยังไง? 3.MRI pelvis 4.Bone scan ก. ล ้างมือก่อนรับประทานอาหาร ข. กินหอยสุก  (Med-GI)ผู ้ป่ วยชาย 60 ปี มาด ้วย easy bruising 2 wk มีประวัตใช ้ ิ ค. กินปลาสุก diclofenac for low back pain for several years ตรวจร่างกายพบ small ง. ใส่รองเท ้า ecchymosis upper leg, mild hepato-splenomegaly, Lab : Hb 11 g/dL, MCV 100, WBC 4,100, Plt 95,000, LFT : AST 37, ALT 150, ALP 130, total bilirubin/direct bilirubin  1.2/0.4, albumin 30, globulin 40 g/L, HbsAg +ve รักษาอย่างไร 2
  • 3. (Med-ID)ชาย 40 ปี มีอาการไอเรือรัง น้ าหนักลด ไข ้ตาๆ chest x-ray มี ้ ่ C. Neutrophil cavity at LUL, AFB 1+ ได ้รักษาเป็ น IRZE 2 เดือน อาการไม่ดขน ตรวจ ี ึ้ D. Complement AFB 2+ จะทาอย่างไรต่อ A. ยาเดิมและนัดติดตามผลการรักษา 1 เดือน  (MED-ID) ผู ้ป่ วยชายอายุ 15 ปี ปั สสาวะลาบากมา 1 วัน เมือ 3 วันก่อนมี ่ ้ ้ B. ส่งเพาะเชือและให ้ยาเดิมจนกว่าผลเพาะเชือจะออก เพศสัมพันธ์กบหญิงขายบริการโดยไม่ได ้ป้ องกัน ตรวจร่างกายพบ ั C. เปลียนยาใหม่ ่ yellowish discharge per urethral meatus ส่ง G/S เป็ น gram ้ D. ส่งเพาะเชือและเปลียนยาใหม่ ่ negative intracellular diplococci with numerous PMN รักษาด ้วยยา E. ยาใหม่(second line drug)... ตัวและยาเดิม ใด A. Doxycycline oral B. Azithromycin oral  (MED-ID)มีไข ้ ปวดหัว มา 1 สัปดาห์ 3 วันก่อน มีตาเหลือง ปั สสาวะออก C. Ceftriaxone IM D. Benzathine pennicilin G IM น ้อยลง PE: mild pale, mild jaundice, mild hepatosplenomegaly, muscle tender  (MED-ID)ชายอายุ 16ปี มาด ้วยปวดศีรษะ และมีไข ้ ตรวจร่างกาย BT 39 LAB: CBC: HB 9.5, WBC 15400(N79%, L20%, E3%) stiffness of neck, Organism? BUN/Cr=95/4 a. S. aureas UA: sp 1.015, pH 6.5, RBC 10-20, WBC 2-3 LFT: AST 210, ALT 150, ALP 71, TB 7.0, DB 5.2, TP7.2, ALB b. N. gonorrheae 3.8 ให ้ DX c. H. Influenzae A. Malaria B. Scrub typhus  (MED ID) ผู ้ชาย 25 ปี มีไข ้ ต่อมน้ าลายโต บวม กดเจ็บ กด Stensen’s C. Leptospirosis duct มี yellow discharge จึงส่ง gram stain D. Acute viral hepatitis ให ้รูปมา เป็ น Gram-negative rod with a characteristic "safety pin" appearance (bipolar staining)  (MED-ID) ผู ้ป่ วยชาย 40 ปี ทางานช่างเรียงพิมพ์ มีอาการไอเรือรัง 2 ้ ้ เดือน ตรวจเสมหะพบเชือวัณโรค จะได ้ยารักษาวัณโรค 4 เดือน ควรทา A. Neisseria meningitides B. Bacteroides fragilis อย่างไร C. Pseudomonas aeruginosa D. Burkholderia pseudomallei A. พักงาน 2 สัปดาห์  (MED ID)กรวยไตอักเสบได ้ ceftriaxone 7วัน หลังจากนันถ่ายเหลวเป็ น ้ B. ใส่หน ้ากากทางาน 2 สัปดาห์ ้ น้ า5ครังคิดถึงเชืออะไร ้ C. ย ้ายงานไปอยู่ในทีทมอากาศถ่ายเทสะดวก ่ ี่ ี D. ติดหลอดไฟทีมแสง UV ่ ี C.difficile E. ให ้ผู ้ร่วมงานกิน INH prophylaxis 3 เดือน  (MED-cardio) ผู ้ป่ วยชาย 50 ปี เหนื่อยง่าย นอนราบไม่ได ้ มา1เดือน ตรวจร่างกาย BP 130/80 , RR 24, PR 80 PMI 6th ICS anterior  (MED-ID) ผู ้ป่ วยหญิงอายุ 30 ปี 2 เดือนก่อนมารพ. ปวดข ้อมือ ข ้อเท ้า axillary line , no mur mur Lung : fine crepitaion at basal both ได ้ยารับประทานดีขน 2 สัปดาห์กอน มีไข ้ เจ็บหน ้าอกซ ้ายเวลาหายใจ ไม่ ึ้ ่ lung ยาใดทีจากการศึกษาวิจัยว่าช่วยลด mortality rate ่ ไอ ไม่หอบ ตรวจร่างกาย: mild pale conjunctiva, swelling interphalanx, decreased breath sound LLL. LAB: CBC Hb10, WBC A. Atenolol 4800(N80%, L20%), Plt 160000 ถาม pathogenesis ว่ากลไกใดเสีย B. Digoxine A. Antibodies C. Diltiazem B. Macrophage D. Amiodipine E. Bisoprolol 3
  • 4. B. IV furosemide  (Med-Hemato)ผู ้ป่ วย femur หักผ่าตัด bleed ไป 2000 ml เลือดgr. AB C. IV Nifedipine แต่รพเลือด gr. AB หมด จะให ้ blood component อะไร D. IV propanolol A whole blood gr A B whole blood gr O  (Med-Cardio)EKG-low voltage QRS manage ยังไง C PRC gr A ก. Pleural tapping D PRC gr B ข. Cardiothoracotomy E PRC gr O ค. Cardiocentesis  (MED-hemato) ชาย 30 ปี chronic alcohol drinker for 10 yr มาด ้วย  (MED Cardio)หญิงอายุ 40 ปี แข็งแรงดีตลอด มาด ้วยเรืองlow grade ่ อ่อนเพลีย 3 เดือน ตรวจร่างกายพบ pale conjunctiva, anicteric, fever และเหนื่อยง่ายมา 14 days, PE : murmur gr.3/6 at apex CBC: glossitis, no hepatosplenomegaly, CBC : Hct 28.5% MCV 125 MCH normal wbc, Echo : Oscillating mass 0.7*0.9 cm at mitral valve 28 MCHC 34 WBC 9,200 (N 60 L 30 B 4 E 1) Plt 90,000 What is MOST likely diagnosis ? the cause of his symptoms? A. MV prolapsed B. Subacute IE a. Chronic alcoholism b. Folate deficiency c. Iron deficiency C. Rheumatic heart disease d. Aplastic Anemia e. MDS D. Cancer endocarditis E. Acute IE  (Med-Cardio) ผู ้ป่ วยมา ER คลาชีพจรไม่ได ้ วัด BP ไม่ได ้ ทา EKG (มีรูป ให ้ดู) เป็ น VT จะทาอะไรก่อน  (MED Cardio)Female, 50 years old, substernal chest pain A adenosine PE: BP 150/110 mmHg, PR 100/min, mild dyspnea,H&L : WNL B defibrillator EKG: ST elevation in II,III,aVF C electrocardioversion Question: After ASA giving – Next step of management? D amiodarone E adrenaline a. Enalapril b. Nifedipine  (Med-Cardio)หญิงอายุ 60 ปี เหนื่อยเวลาทางานบ ้าน ต่อมาเหนื่อยมาก c. Propanolol ขึนเรือยๆ ้ ่ d. Furosemide BP 100/80, RR 30/min, PR 100/min regular, small volume, slow up e. Isosorbide dinitrate stroke carotid pulse, PMI at 5th ICS, 3 cm lateral to MCL, normal S1, decrease S2, bilateral fine crepitation ผู ้ป่ วยน่าจะเป็ นโรคอะไรมากทีสด่ ุ  (Med-Endo)ผู ้ป่ วยชายอายุ 22 ปี ปั สสาวะบ่อยมา …. วัน ปากแห ้ง จึงดืม ่ A. AS น้ ามาก ๆ ไม่ปวดหัว ไม่มตามัว น้ าหนักเท่าเดิม จะส่งตรวจอะไรเพือช่วยใน ี ่ B. MS การวินจฉั ย (โจทย์มเท่านี้จริงๆ) ิ ี C. constrictive pericarditis A. Urine sugar  (Med-Cardio)ผู ้ป่ วยชาย อายุ 60 ปี มี sudden onset ของ chest pain B. Urine specific gravity BP แขนซ ้าย 74/55 แขนขวา 170/110 PR 60/min จะ management C. Plasma glucose อย่างไร D. Serum Creatinine A. oral Enalapril E. Serum electrolyte 4
  • 5. ่ (Med-Endo)หญิงอายุ 55 ปี ใจสัน ร ้อนง่ายมา 6 เดือน ตรวจร่างกายPR  (Med-Skin) ผู ้ป่ วยมีตมคล ้ายลมพิษเป็ นๆหายๆมา 3 เดือน 1 wk ก่อน มี ุ่ 100/min , solitary thyroid nodule ตรวจthyroid scan เป็ น hot thyroid ตุมน้ าใสขึน PE พบ multiple vesicle (bullous) diameter 2-4 cm, on ่ ้ nodule ผู ้ป่ วยปฏิเสธการผ่าตัด นอกจากจะให ้PTUแล ้วจะให ้อะไรอีก erythematous plague on trunk and upper thigh ถามว่า Dx อะไร a. Diazepam A urticarial valgaris b. I131 B Bullous pempigoid c. Propanolol C Pempigous valgaris d. Lugor solution D e. Methymazole E TEN  (Med-Skin)ผู ้ป่ วยหญิงอายุ 25 ปี มาด ้วยผืนแดงขึนบริเวณหน ้าผากและ ่ ้  (MED-endo) หญิงอายุประมาณ 60 ปี (มัง) ซึมลง ไม่กนมา 2 สัปดาห์ มี ้ ิ ลามเป็ นตุมน้ าใสอย่างรวดเร็วมา 1วัน ตรวจร่างกายพบผืนแดงและตุมน้ าใส ่ ่ ่ ้ ประวัตปวดเข่าเรือรัง ซือยากินเองตลอด ตรวจร่างกาย Drowsiness, ิ ้ บริเวณหน ้าผากและเปลือกตาซ ้ายบวมแดง ผู ้ป่ วยมีโรคประจาตัวอะไร Motor เท่ากัน 2 ข ้าง, BP 140/80 ท่านอน, 100/70 ท่านั่ง, PR 80/min ตรวจเลือด plasma glucose 140, Na 115, K ปกติ(จาค่าแน่นอนไม่ได ้), a. HIV Cl 85, CO2 20, b. SLE BUN ปกติ, Cr 1, Serum osmol 240, Urine osmol 500, Urine Na 40 จะส่งตรวจอะไรต่อ  (Med-Skin)ผู ้ป่ วยชาย 65 ปี มาด ้วยผืนคล ้ายผืนลมพิษเป็ นๆหายๆมา 3 ่ ่ เดือน ตรวจร่างกายพบ tense bleb 2-4 cm on erythematous plaque A. Urine VMA วินจฉั ยอะไร ิ B. Serum cortisol A. urticarial vasculitis B. bullous pemphigoid C. pemphigus vulgaris D. erythema multiforme  (MED-Endo)ชายอายุ 45 ปี ตรวจพบว่าเป็ น DM มา3 เดือน FBS 220, E. toxic epidermal necrolysis HbA1c 8.5 ,TG 230 ,choles 210 , น้ าหนัก80 Kg , สูง165 Cm จะstart ยาเบาหวานอะไร  ผู ้ป่ วยชาย อายุ 20 ปี คันบริเวณง่ามนิวมือ ขูดผิวหนังไปตรวจด ้วยกล ้อง ้ A. Metformin จุลทรรศน์กาลังขยายตา พบลักษณะดังภาพ ่ B. Glybenclamide C. Rapeglenide D. Insulin E. Arclabose  (MED Endo)ผู ้ป่ วยหญิง 40 ปี มีก ้อนทีคอมา 9 เดือน ก ้อนไม่โตขึน ตรวจ ่ ้ ร่างกายพบ thyroid nodule 3 cm, firm consistency ทาอะไรต่อ A. thyroid scan B. thyroid uptake C. ultrasound D. antimicrosomal anyibbody E. fine needle aspiration  (MED Endo)หญิง 16ปี มาด ้วย Abdominal pain(clinical DKA), initial MX? A. IV 0.9% NaCl การรักษาทีเหมาะสมกับผู ้ป่ วยรายนี้ ่ B. IV NaHCO3 5
  • 6. A. Antharcin 10 ปี X-Ray ปอดพบ hyperairation, falttern diaphragm, Infiltration B. Sulfur Ointment ที่ upper lung ทังสองข ้าง ้ C. Benzyl Oxydase D. Coal tar ointment a. COPD E. Benzylgamma hexachloride b. Pulmonary TB  (Med-Pulmo) ชาย40ปี ทางานโรงพิมพ์ ได ้รับการวินจฉั ยว่าเป้ นวัณโรค ิ  (MED-skin)ชาย 25 ปี มีผนแดง คัน หนา ทีหนังศีรษะ มา...........เดือน ื่ ่ ่ ้ ปอด ให ้ IRZE 4month จะทาอย่างไรเพือป้ องกันการแพร่เชือให ้คนอืน่ ไม่มอาการผิดปกติอย่างอืน ตรวจร่างกาย small & large erythematous ี ่ 1.Isolates 2 wks scaly plaque at scalp 50 % , yellowish spotting at nail ;Diagnosis ? 2.ใส่ Mask 2 wks A. Tinea capitis 3. ให ้ทางานในทีอากาศถ่ายเท ่ B. Psoriasis 4.ให ้เพือนร่วมงานกิน INH 3 month ่ C. Seborrheic dermatitis 5.ให ้เปลียนหลอดไฟทีทางานเป้ นแสง UV ่ ่  (Med-Pulmo)ชายทางานโรงงานย่อยหิน ไอเรือรัง เหนื่อยง่าย มา 1 ปี มี ้ cxr มาให ้ diag  (Med)ทหารอายุ 20 ปี ถูกนาส่ง ER เนื่องจากเป็ นลมขณะกาลังฝึ กทีกลาง ่ ก. TB แดดเป็ นเวลานาน ตรวจร่างกาย T 42 , PR 130 , RR 30 , BP 140/90 ข. Silicosis mmHg. unconscious and occasional seizure What is the risk factor ค. Sarcoidosis for this patient?  (Med-Pulmo)ชายอายุ 20 ปี มาด ้วย suddenly develops chest pain 1. Male gender and progressive dyspnea within 3 hours. PE: decreased breath 2. High humidity sound on the left lung and increased tympany on percussion 3. Previous seizure ถาม emergency management? 4. young age group a) ET tube intubation b) Thoracocentesis 5. genetic predilection c) Oxygen support  ชาย50 ปี underlying HT มาด ้วยปวดทั่วๆ ท ้องมา1วัน ขาไม่มแรงเดิน ี d) ICD ไม่ไหว e) จาไม่ได ้อ่ะ ตรวจ PE PR arm = 110 pulse ทีขาคลาไม่ได ้ BP 90/60 ่ What is initial management  (Med-Pulmo)ผู ้ป่ วยชาย อายุ 70 ปี มีอาการเหนื่อยง่ายเวลาขึนบันได ้ A. Emergency sugery เป็ นๆ หายๆ มา 10 ปี มีไอมีเสมหะ สูบบุหรีวนละ 1 ซอง มา 30 ปี ตรวจ ่ ั B. Dopamine ร่างกายพบ C. IV resuscitation Lung : Barrel shape, decrease breath sound both lungs. D. Whole blood ่ ่ ข ้อใดเป็ นการรักษาทีชวยให ้คุณภาพชีวตดีขน ิ ึ้ E. Cut down วัด CVP A. Bronchodilator B. Ipratropium bromide C. Inhalation corticosteroid  (Med-Pulmo) ผู ้ป่ วยชายอายุ 55 ปี มาด ้วยอาการเหนื่อยง่ายมากขึนเวลา ้ D. Pulmonary rehabilitation ทางาน มีไอเล็กน ้อย ทางานทีโรงงานทาครกหิน สูบบุหรี่ วันละ 1 ซองมา ่ E. Long term oxygen therapy 6
  • 7. (MED Pulmo)ชายอายุ 35 ปี ทางานอยู่ในโรงงานเกียวกับแร่ใยหิน มี ่  (MED Rheumato)ญ อายุ 40 ปี มีอาการปวดช ้อมือ ข ้อนิว ทัง 2 ข ้าง รู ้สึก ้ ้ อาการไอเรือรังมาประมาณ 1 ปี chest x-ray พบ diffue infiltration ที่ ้ เหมือนมีไข ้ตาๆ ตรวจร่างกาย มี Tender&Swelling ที่ 2nd MCP,2nd ่ ปอด 2 ข ้าง จงให ้การวินจฉั ย ิ 3rd 4th PIP ข ้างขวา และ 3rd MCP 3rd 4th PIP ข ้างซ ้าย A. Silicosis ..... B. Aspergillosis  (Med-Toxico)ชาย 20 ปี พีสาวพามามาที่ ER ด ้วยปั ญหายาเสพย์ตด มี ่ ิ C. Asbestosis อาการปวดกระดูก และกล ้ามเนื้อ ตัวงอร ้องไห ้ ยอมรับว่าตนเองใช ้สาร เสพย์ตดหลายชนิดปนกัน ตรวจร่างกาย Temp 37 PR 87 RR 20 ถาม ิ  (MED Pulmo)A previously healthy 60-year-old man presents with Manageไง left-side substernal pain then productive cough with rusty 1. Oral clonidine sputum, no history of smoking and he denies HIV risk, chest pain 2. Oral naltrexone is sharp, locate on left, no radiate PE: T 39C RR 26/min BP 90/60 3. Oral methadone P90 Chest: dullness on percussion and decrease breath 4. IM Haloperidol sound(โจทย์ตรงตรวจร่างกายมันมีอก แต่มแต่พวก negative finding อีก ี ี 5.IV Naloxone ่ 2 อย่าง ซึงจาไม่ได)้ ให ้ Dx A. Pericarditis  (Med-Toxico)ชาย 35 ปี โดนงูกดทีมอขณะตัดกิงไม ้ทีสวนยางพารา 3 ั ่ ื ่ ่ B. TB pleuritis ่ ชัวโมงก่อนมารพ. มีปวดบริเวณมือถึงแขน PE: ecchymosis and bullous ทีมอ ถามว่างูชนิดใดทาให ้เกิดอาการ ่ ื C. Influenza pneumonia a) งูเห่า D. Obstructive pneumonitis b) งูกะปะ E. parapneumonic effusion c) งูแมวเซา d) งูสามเหลียม ่  หญิง 60 ปี underlying DM, HTN, DLP มารพ.ด ้วยอาการ Dyspnea and e) งูทับสมิงคลา Diaphoresis . What is the most likely diagnosis ?  (MED-toxico) ผู ้ป่ วยหญิง 20 ปี ทะเลาะกับเพือนชาย น ้อยใจจึงกินน้ ายา ่ A. Panic attack ล ้างห ้องน้ า เพือนชายพามารพ.ทันที เอาขวดน้ ายาล ้างห ้องน้ ามาด ้วย ่ B. Pulmonary embolism พบว่าประกอบด ้วย HCl 15% vital sign ปกติด ี มี mild erosionทีปากและ ่ C. Acute myocardial infarction กระพุ ้งแก ้มเล็กน ้อย ผู ้ป่ วยรู ้สึกตัว บ่นว่าปวดแสบในช่องปากและลาคอ D. Pneumonia กลืนน้ าลายไม่ได ้และปวดท ้อง ถาม ข้อห้าม การรักษาในผู ้ป่ วยรายนี้ คือ ข ้อใด  (MED-nephro) ชาย 50 ปี เพลีย คลืนไส ้ อาเจียน ปั สสาวะน ้อยลง 2 วัน ่ a) ดืมน้ ามากๆ ่ ตรวจร่างกายพบว่ามี dyspnea, edema, lungs-rales BLL, BUN 80 Cr b) กิน sucralfate 5.7 นอกจาก UA แล ้วจะ investigate อะไร c) กิน paracetamol d) กินสารละลาย NaHCO3 a. Plain KUB b. U/S Kidney c. Renal Scan d. IVP e) ส่ง endoscope ใน 48 ชัวโมง ่  (MED-Rheumato) หญิง 25 ปี มีอาการปวดข ้อนิว เป็ นมากตอนเช ้า มี ้ painless oral ulcer, DCT-positive. Diagnosis? a. Reiter b. Reactive Arthritis c. SLE d. Rheumatoid Arthritis 7
  • 8. (MED Tox)ชายอายุ 26 ปี ญาตินาส่ง รพ.ด ้วย หมดสติ ปลุกไม่ตนมา 2 ื่  (MED)ผู ้ป่ วยชาย 20 ปี มาด ้วยท ้องโต ขาบวมมา 3 เดือน ตรวจร่างกายพบ ชม สงสัยว่ากินยาเกินขนาด PE: semicoma, Pupil dilate 4 mm both sign chronic liver disease +ve, ascites 2+, mild jaundice Lab : Hct eyes, ปากแห ้ง, ผิวแห ้ง, decreased bowel sound EKG: wide QRS, 28%, WBC 3,500, AST 230, ALT 100, ALP 30, total protein 80 g/L, albumin 30 g/L, anti-HBc +ve, HbsAg –ve, anti-HBs –ve, anti-HCV Prolong QT คิดว่าน่าจะเกิดจากยาใด –ve, ANA 1:40 a) codeine A. serum ferritin B. protien electrophoresis b) hyoscine C. HCV PCR D. anti-smooth muscle antibody c) amitryptyline d) amphetamine  (MED) หญิงไทย60ปี เป็ นCA Stomach ระยะสุดท ้าย แพทย์ไปเยียม ่ e) CPM บ ้าน พบว่าผอมมาก หนังหุ ้มกระดูก ญาติบอกผู ้ป่ วยทานอาหารได ้น ้อย มาก ขอปรึกษาเรืองการให ้อาหารกับผู ้ป่ วย ข ้อใดเหมาะสมสุด ่  (MED TOX) ทหารทีชายแดนภาคใต ้ปะทะกับผู ้ก่อการร ้ายทุกวัน มี ่ 1.Oral feeding as tolerance อ่อนเพลีย ซีด ปวดท ้อง ตรวจร่างกายพบว่ามีแถบสีดาทีเหงือกด ้านบน ่ 2.No feeding เกิดจากสารพิษใด 3.Jejunostomy tube feeding A.ปรอท B. ตะกัว ่ 4.partial parenteral nutrition C.ทองแดง D.แมงกานีส 5.Total parenteral nutrition E.คาร์บอน  (MED-ID)หญิง อายุ 50 ปี ทาสวน ปวด คันข้ อเท้ า ตรวจร่างกายพบ erythematous, tubular lesion เกิดจากเชื ้อใด  (MED) ผู ้หญิง แข็งแรง no U/D มาด ้วยปวดขาขวามา 5 วัน, กินยาคุมมา 3 เดือน ตรวจร่างกาย swelling and tender Rt. leg ให ้ Dx 1. Toxocara a) cellulitis 2. Trichinella b) lymphedema 3. Stongyloides c) deep vein thrombosis 4. Gnathostomiasis d) osteolitis 5. Angiostrongyrus e) arterial occlusion  (MED)ผู้ ป่วย post op. cardiac surgery มีอาการสับสนโวยวาย เห็นพญายม จา  (MED )หญิงอายุ 45 ปี มาด ้วย central upper abdominal pain 2 days ญาติไม่ได้ พูดว่าอย่ามาหลอกกันเลย what is immediate management ,PE:fever, tender epigastrium, Past history: no smoking,no trauma,no alcoholic , LFT: AST 146,ALT 160,TB 3.0 DB1.9; A. Diazepam Amylase 800, CXR: WNL Most appropriate investigation? B. Midazolam C. Fluoxetine A. U/S upper abdomen D. Haloperidol B. MRI E. Nortriptyline C. CT D. ERCP 8
  • 9. (MED) ชายอายุ 70ปี clinical dementia ,มีอาการหลงลืม กินยาเกินขนาดเพราะจา 2 Corneal ulcer ด้ านซ้ าย ไม่ได้ ว่ากินไปแล้ ว no neuro deficit ถามว่าmost factor ที่จะทาให้ อาการแย่ลงคือ 3 ชาหน้ าซีกซ้ าย อะไร 4 Sensorineural hearing loss หูซ้าย A. ไขมันในเลือดสูง 5 ชาลิ ้นด้ านหน้ าซ้ าย B. เส้ นเลือดสมองตีบ  (MED)ชายไทยอายุ 50 ปี เป็ น Hypertension 5 ปี ได้ รับยา atenolol มาด้ วย C. อาชีพก่อนเกษี ยณ sudden onset chest pain วัด BP right arm 140/90 , left arm 70/40 ให้ ยา D. ความเครียดของผู้ดแลู E. ความสัมพันธ์ ของคนในครอบครัว รักษาตัวใดอันดับแรก (ตอบข้ อ 2) A. sublingual nitroglycerine  (MED)ผู้ ป่วยชายอายุ 50 ปี เหนื่อยหอบ ตัวบวม ปั สสาวะออกน้ อย มาสองวัน urine B. nitropusside IV output 50 Bun/Cr 50/5.6 CXR pulmonary congestion นอกเหนือจาก C. propanolol D. sublingual nifedipine UA ให้ Initial investigation เพิ่มเติม Pediatrics A. Film KUB B. Ultrasound KUB  ้ (Ped-Hemato) เด็กชาย 3 ปี เลือดออกในข ้อบ่อย ชาง่าย มีพชายและลุง ่ี มีอาการคล ้ายกัน CBC ปกติ Plt 170000,APTT prolong, bleeding time C. Renal scan ปกติ, PT ปกติ, D. IVP TT ปกติ Dx? E. CT abdomen - Hemophilia - Hereditary thrombocytopenia  (MED)หญิงอายุ 55 ปี ปวดเข่ามา 2 ปี ตรวจร่างกายไม่พบ joint deformity ชัดเจน - Hereditary platelet dysfunction แต่พบใบหน้ ากลม มี trunkal obesity, supraclavicular fat pad สาเหตุที่ทาให้ เกิด - Von Willebrand disease - Congenital factor VII deficiency อาการข้ างต้ น คือ a. Adrenal adenoma  (Ped-ID)เด็กหญิง 14 ปี ปั สสาวะแสบขัด คันช่องคลอดและทวารหนักมา b. pituitary microadenoma 1 เดือน ไม่เคยมีเพศสัมพันธ์ ตรวจร่างกาย erythematous rash at c. pituitary macroadenoma vagina & perigenital area , yellowish discharge ถามการส่งตรวจทีม ี ่ d. ectopic ACTH ประโยชน์ในการวินจฉั ยโรค ิ e. exogenous glucocortocoid A. urine culture B. scottape technique  (MED)ผู้ ป่วยชายอายุ 40 ปี มาด้ วย ปากเบี ้ยวด้ านซ้ ายและตาซ้ ายปิ ดไม่สนิทมา 1 C. darkfield microscopic D. discharge Gram stain สัปดาห์ clinical ที่พบร่วมได้ ในผู้ป่วยรายนี ้คือ E. discharge KOH preparation 1 แขนขาขวาอ่อนแรง 9
  • 10. (PED ID)หญิง 15 ปี มีไข ้ ไอแห ้งๆ ตรวจร่างกาย BP 100/60mmHg, PR C. ให้ ซกผ้ าปูที่นอนเองทุกครั ้งที่ปัสสาวะราด ั 100/min, T 37.9c, RR 25/min fatigue, malaise, nasal congestion, D. Imipramine mild injected pharynx and tonsils, cervical lymph node โต a. influenza b. acute sinusitis E. พบจิตแพทย์ c. pneumonia d. pharyngitis e. infectious mononucleosis  เด็กสามขวบ ไม่มเขียว พัฒนาการปกติหมด หนัก 14kg สูง 95cm ข ้อใด ี ้  (Ped-Dev) เด็ก 1 ขวบ 6 เดือน แม่พามาเรืองพูดช ้า ยังไม่พูดคาทีม ี ่ ่ ถูกต ้อง ความหมาย เรียกไม่คอยหัน ชอบดูทววนละ 2-3 ชม. ชอบเล่นคนเดียว ไม่ ่ ี ี ั a. ricket ี้ สบตา ไม่ชบอกความต ้องการ แม่เป็ นคนเลียงเอง Dx? ้ b.hypopituitary - Selective mutism c.hypothyroid - Hearing impairment d. …….dysplasia - Autism spectrum disorder e.normal variant - Inappropriate childbearing - Developmental language disorder  (PED-newborn) Newborn, preterm, low birth weight มีอาการเหนื่อย O2 sat 86% PaO2 58 mmHg, plain film พบ ground glass both lungs  (PED-dev) มารดาพาลูกชายอายุ 8 ปี มาปรึกษาท่านด ้วยปั ญหาการเรียน ถาม Management และพฤติกรรม โดยเล่าว่า ผู ้ป่ วยมีอาการหงุดหงิดง่าย เกรดเฉลีย 2.8 วิชา ่ ภาษาไทยได ้เกรด 0 วิชาอืนได ้เกรด2-3 ครูภาษาไทยบอกว่าผู ้ป่ วยไม่ ่ a. PPV b. corticosteroid c. mechanical ventilation d.CPR ตังใจเรียน แต่ครูวชาอืนบอกว่า ผู ้ป่ วยตังใจเรียนดี ผู ้ป่ วยทาการบ ้านวิชา ้ ิ ่ ้ คณิตศาสตร์ได ้จนเสร็จ แต่ทาการบ ้านภาษาไทยไม่ได ้ อ่านหนังสือไม่ได ้ ความสัมพันธ์กบเพือนปกติ พัฒนาการปกติ diagnosis is..? ั ่  (PED-newborn) เด็ก 2800g. ยาว 48 cm. มี cleft lip and palate, bilateral cryptorchidism, penile length 1 cm, glucose 30. What is a. language disorder b. learning disorder diagnosis? c. conductive disorder d. ADHD a. hyperinsulinemia b. hypopituitarism c. adrenal insufficiency d. growth hormone deficiency e. inborn error of metabolism  (PED-dev) at well-baby clinic, 18-month-old baby ร ้องไห ้เวลาไม่ให ้ กินเอง จับช ้อนกินข ้าวได ้ วาง cubic 4 ชินได ้ หันตามเสียง พูดแต่คาว่า ้ mama. What is the most concerned problem in this children?  (Ped-Newborn) เด็ก newborn 48 ชม. เหลือง microbillirubin 18 Hct 38% Aniso2+ Poikilo1+ Fragmented2+ Spherocyte2+ Heinz and a. language b. fine motor c. gross motor Inclusion body negative ถามว่าเกิดจาก 1.ABO incompatibility d. behavior e.social 2.RH incompatibility 3.G6PD  (PED-dev)เด็ก 7 ปี ปั สสาวะราดที่นอนมาตลอด กลางคืนปั สสาวะ 3-4 ครั ้ง กลั ้น 4.Hb-H disease 5.Physiological jaundice ปั สสาวะได้ 5-10 นาที ตรวจร่างกายปกติ ผลการตรวจปั สสาวะปกติ จะแก้ ไขอย่างไร A. ใส่ผ้าอ้ อมสาเร็จรูปตอนกลางคืน B. ให้ แม่ปลุกตอนกลางคืนเพื่อปั สสาวะ 10
  • 11. (Ped-Immuno)เด็ก 8 เดือน recurrent bacterial pneumonia มีพชาย ี่  (PED-pulmo) เด็กอายุ 5 ขวบ 3วันก่อนมีไข ้ ไอ หายใจเร็ว PE: เสียชีวตเมืออายุ 9 เดือน ด ้วย recurrent pneumonia ผู ้ป่ วยไม่เคยติด ิ ่ expiratory wheezing, occasional crepitation CXR: mild ้ เชือ virus, fungus ถามว่า ผู ้ป่ วยมีความผิดปดติของ cell ใดในimmune hyperaeration, parabronchial infiltration ให ้ bronchodilator ไม่ดขน ี ึ้ system ให ้ DX A. T Cell a) Acute epiglossitis B. B Cell b) Tracheobronchitis C. Combine T and B cell c) Acute Bronchiolitis D. Phagocytic cell d) Bronchopneumonia e) Acute Asthmatic attack  (PED-allergy) แม่มลกน ้อย กลัวว่าลูกจะเป็ นภูมแพ ้ เพราะมีประวัต ิ ี ู ิ ครอบครัวเป็ น allergy จะ advice อะไร  (PED-neuro) ผู ้ป่ วยอายุ... มีไข ้ 3 – 4 วัน......... ไม่มี meningeal sign LP : Mononuclear cell 83% Protein 40 % Glucose 60 (Blood a. air filter b. mite-proof mattress c. no pet glucose 80 ) 1. Brain abscess 2. Viral meningitis d. soy milk e.exclusive breast feeding  (Ped-Prevention)เด็ก 9 เดือน ไม่ควรใช ้อะไรเพือป้ องกันอุบัตเหตุ ่ ิ  (PED-cardio) เด็กหญิงอายุ 6 ปี มีความดันสูง ตรวจร่างกายพบ BP 1.Rattler 140/80 mmHg, SEM gr II LUPSB, pulse แขนแรงกว่าขา ผู ้ป่ วยมี 2.High chair congenital anomaly ใด 3. รถเข็น 4. รถหัดเดิน a. Coarctation of Aorta b. ASD c. PDA d. TGA 5.จุกนมหลอก  (Ped-GI)เด็ก 9 เดือน มาด ้วย watery diarrhea ตรวจร่างกายพบ dry  (PED-Prevention)เด็กอายุ 5 ปี ตกบันได ผู ้ปกครองพามาโรงพยาบาล lips ทา stool exam : yellowish mucous stool เจอ WBC 5-10 จะส่ง ตรวจร่างกายพบรอยฟกช้า ตามตัวซึงมีทังรอยเก่าและรอยใหม่ จะทา ่ ้ investigationใดเพือdiagnosis ่ อย่างไรต่อกับผู ้ป่ วยเด็กรายนี้ (ตอบข ้อ 3) A. Stool culture A. ให ้กลับบ ้าน B. Stool reducing sugar B. ปรึกษาสังคมสงเคราะห์ C. Stool PCR for Rota virus C. รับไว ้ในโรงพยาบาล D. Stool occult blood  ่ (PED-child abuse) เด็กชายอายุ 8 ขวบ มีประวัตตกบันไดมา 1 ชัวโมง ิ  (Ped-Pulmo)เด็ก 5 ขวบมี sudden onset of dyspnea ไอ ฟั งปอดพบ ก่อนมา รพ. ตรวจร่างกายพบ รอบก ้นมีผน ตามร่างกายมีรอยฟกช้าทังเก่า ื่ ้ decreased breath sound ข ้างขวา จะทาอะไรต่อไป ใหม่กระจายทั่วร่างกาย น้ าหนักตากว่าเกณฑ์ พัฒนาการตามวัย ท่ านจะให ้ ่ a. rigid bronchoscope การักษาอย่างไร b. CXR c. fiberoptic bronchoscope ก. admit d. thoracotomy e. ICD ข. ส่งปรึกษานักโภชนาการ ค. ส่งปรึกษานักสังคมสงเคราะห์ ง. ให ้ยาแล ้วให ้กลับบ ้าน 11
  • 12. (PED-Endocrine)เด็กหญิง 5 ปี polydipsia polyuria น ้าหนักลด 5 kg ใน 2 เดือน o muscle fiber o peripheral nerve fiber มาที่ ER ด้ วย agitation, dehydration and kussmual breathing investigation: o anterior horn cell blood sugar 520, Na 126, K 6.5, CO2 8mmol/l, urine ketone 4+ ถาม Initial management  (PED)A 10 yr olds with uncontrol asthma present with difficult breathing a) NSS , chest pain after several cought , PE: expiratory wheezing , diminish b) 3% NaCl breath sound left lung, heart sound to right side Dx c) 7.5% NaHCO3 1. Atelectasis d) IV insulin 2. pleural effusion e) rectal Kayexalate 3. pneumothorax 4. PE 5. acute bronchospasm  (PED-Child dev.)เด็ก อายุ 12 เดือน ผ่าตัด cleft lip แล้ ว เหลือ cleft palate ที่ soft palate พ่อแม่กงวล กลัวว่าการผ่าตัดจะเป็ นอันตราย ท่านเป็ นแพทย์จะบอกพ่อแม่ว่า ั OB-GYN  (OB-GYN) 30 years-old G3P2 GA 38 wks. Presented with Vaginal ถ้ าไม่ผ่าตัดจะส่งผลเสียอย่างไรมากที่สด ุ bleeding & Abdominal pain. She has unremarkable ANC history. PE: V/S – stable, fundal height 40cm above umbilicus, o เสียงแหบ Longtitudinal lie, Cephalic Presentation, Contraction every 2 mins o เสียงแหลม 50sec in duration, Fetal HR 140/min reactive. o ติดอ่าง U/S- Retroplacental 4x3 hematoma at fundus PV- Cx dilatation 8cm, effacement 80%, Intact membrane, station o พูดเสียงขึ ้นจมูก +2 o พัฒนาการด้ านการพูดช้ า What is the most appropriate Mx? 1. Continue spontaneous delivery  (PED)เด็กชาย 10 ปี มีอาการอ่อนแรงขา 2 ข ้างมา 3 วัน เมือ 1 สัปดาห์ ่ 2. IV Oxytocin ก่อนมีไข ้ น้ ามูกไหล PE: bladder distention,muscle power grade 0 , 3. Amniotomy ชาตังแต่ใต ้สะดือลงมา, BBK:positive both side,deep tendon reflex: ้ 4. V/E absent Which of the following is most likely diagnosis? 5. C/S A. Poliomyelitis  (OB-GYN)หญิงอายุ 50 ปี 1 wk PTA ไปทา F&C มาเนื่องจากมี B. Transverse myelitis abnormal vaginal bleeding มา 4 wk PTA หลังทามีไข ้ ปวดท ้องที่ C. Viral myositis suprapubic area ตรวจภายใน พบ foul smell and tender at D. Collagen vascular disease suprapubic area เกิดจากอะไร E. GBS 1. salphingitis  (PED)เด็กอายุ 7 ปี เริ่มเดินลาบากมา 4 ปี slow verbal , wadding gait, cranial 2. endometritis nerve intact, slow reflex ถามว่าผิดปกติที่ไหน 3. TOA 4. reetained piece of endometrium 12
  • 13. (OB-GYN) หญิงไทย 39ปี มีตกขาวสีเหลืองมา 3-4เดือน PV; yellowish d) F/E discharge with inflammed cervix with erosion ถามเกิดจากสาเหตุใด e) C/S 1.HSV 2.HPV  ผู ้ป่ วยหญิงอายุ38ปี เป็ นเบาหวานตังครรภ์ GA 30wks. มาด ้วยเจ็บครรภ์ ้ 3.N.gonorrhea คลอดและปั สสาวะบ่อย ตรวจพบมดลูกอยู่ระดับลินปี่ คลาส่วนของทารกไม่ ้ 4.C.trachomatis ชัด FHS 152/min Uterine contraction 1ครังใน30นาที PV: cervix ้ 5.Gardnerella vaginalis dilated 1cm , effacement 50% อยากทราบว่าสาเหตุททาให ้เกิด ี่ ลักษณะในผู ้ป่ วยรายนี้คอ ื  (OB-GYN)หญิงหลังคลอดบุตร ปวดเต ้านมซ ้าย มีบวมแดงร ้อน ตรวจ ร่างกายพบ erythematous A. Macrosomia crack skin around nipple and areola. จะให ้ antibiotics อะไร B. Twin pregnancy A. Cloxacillin C. Myoma uteri B. Erythromycin D. Polyhydramnios C. Ceftriaxone E. Urinary tract infection  (OB-GYN)หญิงไทยอายุ 28 ปี G1P0 GA 38 wk มาด ้วย labor pain จึง  (OB-GYN) Pregnancy with labor pain, transverse lie ทาไรอะต่อ drip oxytocin ปวดที่ suprapubic area BP 90/40 mmHg PR 110 bpm a. Observe มี bleeding per vagina b. C/S a. sedate b. หยุดให ้ oxytocin  (OB-GYN) หญิง 25 ปี G1P0 GA 12 wk มีอาการคลืนไส ้อาเจียนมากมา ่ c. เพิม rate oxytocin ่ 1 สัปดาห์ ตรวจ uterus คลา fundal height 2/3 > suprapubic จะให ้ d. เปลียน position ่ e. explore laparotomy การรักษาอะไรก่อน A. IV fluid  ่ ่ (OB-GYN)หญิง 45ปี มีก ้อนตุงทีชองคลอด ตรวจร่างกาย พบ anterior vaginal wall ลงมาที่ hymen, cervix เลย hymen มา 2 cm จะตรวจอะไร  (OB-GYN) หญิงอายุ 30 ปี G3P1A1 GA 38wk มีเลือดออกทางช่อง เพิมเติม (ข ้อสอบเก่า) ่ คลอดปริมาณมาก ท ้องแข็งเกร็งเป็ นพักๆทุก 5 นาที ลูกชายอายุ 5 ปี ผ่า 1.whiff test คลอด มีประวัตขดมดลูก 3 ปี ก่อน PR100 BP 110/70 FH=3/4 เหนือ ิ ู 2. fern test สะดือ ballotment at fundus FHS 140 เป็ นอะไร 3. Q- tip test a. Vasa previa 4. Pap test b. Placenta previa 5. Stress test c. Abruptio placenta d. Uterine rupture  (OB-GYN)หญิงอายุ 25 ปี G3P2 GA 30 weeks มาด ้วยเจ็บท ้องคลอด e. Massive bloody show และมีน้ าเดิน PV: Cervix fully dilated, ถุงน้ าคร่าแตกแล ้ว, เด็กอยู่ในท่า mentoposterior, station 0, FHR 100 bpm, regular ถามว่าจะทาอะไร?  (OB-Gyn)ผู ้หญิงอายุ 28 ปี อายุครรภ์ 28 สัปดาห์ ตรวจ VDRL reactive a) Spontaneous vaginal delivery titer = 1:32, FTA-ABS positive ให ้การรักษาเป็ น Benzathine b) Augmentation of labor penicillin 2.4 ล ้านหน่วย ทางกล ้ามเนื้อเป็ นเวลา 3 สัปดาห์ ต่อมาตรวจ c) V/E ซ้าตอน อายุครรภ์ 32 สัปดาห์ VDRL = 1:8 ท่านจะให ้การรักษาที่ เหมาะสมทีสดอย่างไรต่อไป ่ ุ 13
  • 14. A. วัด VDRL ซา ตอนหลังคลอด 6 สัปดาห์ d. Twisted ovarian B. ฉีด Benzathine penicillin ซ้าในขนาดและเวลาเท่าเดิม e. Rupture ectopic pregnancy C. ฉีด Benzathine penicillin 2.4 หน่วย ครังเดียว ้ D. ให ้penicillin หยดทางหลอดเลือดดา 1.2 ล ้านหน่วย ระยะเวลา____  (OB-GYN)หญิง 25 ปี G1P0 GA 35 wk ประสบอุบติเหตุรถชน มาที่ ER V/S ั E. เปลียนยาเป็ น erythromycin วันละ 2 กรัม 30 วัน ่ ปกติ ตรวจพบมดลูกแข็งตัวตลอดเวลา Cervix dilate 5 cm, FHS 140 bpm ่ ่  (OB-Gyn)ผู ้ป่ วยหญิง 45 ปี มาด ้วยก ้อนตุงทีชองคลอด 1 ปี ตรวจร่างกาย พบผนังช่องคลอดด ้านหน ้าหย่อนมาที่ hymen ปากมดลูกยืนเลย hymen ่ ตรวจเพิ่มเติมแล้ ววินิจฉัยเป็ น Abruptio placenta จะ management อย่างไร มา 2 cm จะต ้องตรวจอะไรต่อ A. Fern test A. Observe uterine contraction B. Pap test B. Tocolytic drug C. Q-tip test C. Dexamethasone D. Stress test D. Oxytocin E. Whiff test E. C/S  (OB-Gyn)หญิงไทย 30 ปี G2P1 เคยตังครรภ์ครังที่ 1 แท ้งตอน 6 wk ้ ้ ครรภ์ 2 v/s Normal ทา Transvaginal U/s พบ sac แต่ไม่พบตัวเด็ก  (OB-GYN)หญิงไทยตั ้งครรภ์ อาชีพกรรมกร อายุ 35 ปี มาด้ วยเรื่องน ้าเดิน 6 มี bleeding per vagina ถามว่าสาเหตุน่าจะเกิดจากออะไร ชัวโมงก่อนมารพ คลอดด้ วยวิธี C/Sเนื่องจาก fetal distress ระหว่างรอ ่ A. Uterine abnormal B. Cervical incompetence คลอดถูกตรวจภายใน 4 ครั ้งใน 6 ชัวโมง 3 วันต่อมาผู้ป่วยมีไข้ Hct 24% ่ C. Ectropic prewgnancy ข้ อใดเป็ นปั จจัยเสียงของภาวะแทรกซ้ อ นข้ างต้ น ่ a. อาชีพ  (OB-Gyn) หญิงอายุ 20 ปี G1 เจ็บท ้องคลอดมา stage 1ปกติ ผ่าน stage 2 ไปแล ้ว มดลูกหดตัวดี, FHS ดี, แรงเบ่งดี, PVมดลูกเปิ ด 8 cm, b. C/S effacement 100 %, station +2, ROT, แต่คลาไม่ได ้ molding & c. PROM caput ภาวะทีวนจฉั ยในผู ้ป่ วยรายนี้คอ ่ ิ ิ ื d. anemia A. mentoposterior e. PV 4 ครั ้ง B. persistent occiput posterior C. deep arrest transverse  (OB-GYN) หญิง 25 ปี G1P0 GA 38wk มีอาการปวดศีรษะ รู้สกว่าลูกดิ ้นน้ อยลง ึ D. hypotonic uterine contraction E. Cephalopelvic disproportion PE BP 180/110mmHg T 37C RR 20/min uterine size = GA proteinuria 2+ DTR 3+ ควรให้ ยาใด  (OB-GYN) ผู้หญิงไทยอายุ 25ปี มีไข้ สง ปวดท้ องมา 10วัน ตรวจร่างกายพบ ู 1. Diazepam abdominal mass, peritonitis, no guarding, cervix excitation pain positive 2. Magnesium sulphate What is most likely diagnosis? 3. Antihypertensive drug a. Tubo-ovarian abscess 4. …….. b. PID 5. ……….. c. Appendiceal abscess 14
  • 15.  (OB-GYN)G1P1 GA 38 wks. มาด้ วยน ้าเดินมา5วัน คลอดปกติ หลังจาก  (Surg-Uro)ชาย 45 ปี 2 ปี ก่อน ตรวจพบ Rt. ureteric stone size 1.5 cm ผู ้ป่ วยปฏิเสธการรักษา 2 วันก่อน ปั สสาวะไม่ออก หอบเหนื่อย U/S พบ คลอด1วันมีไข้ T.39.5 พูดจาสับสน จาสามีไม่ได้ เห็นผีบอกว่าอย่ามาหลอกกันเลย Rt. severe hydronephrosis, Lt. moderate hydronephrosis with Lt. จงวินิจฉัย renal stone 2 cm LAB: Na 130, K 7, Cl 100, CO2 12, Cr 9.8 What is the most appropriate management? 1. Dilirium a. ESWL Lt. renal stone 2. D. Postpartum psychosis b. ESWL Rt. Ureteric stone 3. Brief psychosis c. Lt. nephrolithotomy 4. Postpartum depression d. Lt. percutaneous nephrotomy 5. Postpartum blue e. Rt. transurethral laser lithotripsy  (Sx URO) ชายอายุ 65 ปี เป็ นเบาหวานมา 10 ปี 3 เดือนก่อนมาพบ แพทย์ด ้วยเรืองปั สสาวะบ่อย ปวดเบ่ง ปั สสาวะไม่พุ่ง ได ้ α-blocker ่ Surgery อาการไม่ดขน วันนี้มาด ้วยปั สสาวะไม่ออกมา 1 วัน PRmild enlarge ี ึ้  (Surg)ชาย 40 เป็ นก ้อนทีหลัง 10 เซนติเมตร บริเวณ lumbrosacral area ่ prostate gland, smooth surface, rubbery และมี loose sphincter ก ้อนไม่ตด skin ไม่เจ็บ ไม่มรอยบุ๋ม ิ ี tone แพทย์พจารณาผ่าตัด TUR-P จะส่งตรวจอะไรเพิมเติมทีสาคัญทีสด ิ ่ ่ ่ ุ A. Lipoma เพือพิจารณาทา TUR-P ่ B. Meningioma A. Plain KUB C. Chordoma B. cystrometry D. Epidermal cyst C. Urethrography D. panoscopy  (Surg)ชายไทยอายุ 50 ปี มีก ้อนโตทีหลังมา 3 ปี ค่อยๆโต ไม่เจ็บ ่ E. transrectal U/S movable , soft sonsistency จงให ้การวินจฉั ย ิ a. lipoma b. epidermal cyst  (Surg) ผู้ป่วยชาย 18 ปี ปวดท้ องรอบสะดือมา 1 วัน 2 hr ก่อนมา รพ. ปวดมากขึ ้น c. melanoma ย้ ายมาปวดที่ RLQ ไม่มีคลืนไส้ อาเจียน มีเบื่ออาหาร ตรวจร่างกายพบ T 37.0c, ่ d. meningocele e. teratoma tender with guarding at RLQ, no rebound, diminished bowel sound, Lab มี WBC 3,000 ( N 60%, L 40% ) ส่ง investigation อะไรในขณะนี ้  (SURGERY) ผู ้ป่ วยหญิงอายุ 40 ปี ผ่าตัดคลอดลูก 2 ครัง มีปวดท ้อง ้ บริเวณตรงกลางเป็ นๆหายๆ ไม่ถ่าย ไม่ผายลมมา 1 วัน PE T 38 C BP A. UA 130/80 mmHg P 100/min RR 18/min มีปากแห ้ง ท ้องอืด bowel B. CT lower abdomen sound ลดลง มี mass ที่ Right lower quadrant abdomen ขอบเขตคลา ั ได ้ไม่ชดเจน Guarding at RLQ CBC Hct 40% WBC 15,000(N 80% L C. diagnostic laparoscopy 20%) จะส่ง Investigation อะไรเป็ นอันดับแรกเพือการวินจฉั ย ่ ิ D. ไม่ต้องทาอะไร รอ 24 hr A. Plain film abdomen E. ไม่ต้องทาอะไร ผ่าตัดเลย B. U/S lower abdomen C. CT scan whole abdomen D. Barium enema E. Colonoscopy 15